You are on page 1of 87

Edited by Foxit Reader Copyright(C) by Foxit Corporation,2005-2009 For Evaluation Only.

Chapter 11

KHIZZER KHAN

Blood vessels
Q.NO.1.A 55 years old man presents with lift sided facial pain, with palpable left temporal artery. Biopsy of the artery reveals fragmentation of internal elastic lamina, with granulomas containing langhan and foreign body giant cells. (annual 2010) a. what is the diagnosis? b. which other condition should be considered in the differential diagnosis if a granulomatous vasculitis involves the aorta? c. list the three pathogenic mechanisms involved in non-infectious vasculitides. a. Giant cell arteritis b. Takaysu arteritis c. immune complex associated vasculitis antineutrophil cytoplasmic antibodies anti-endothelial cell antibodies Q.NO.2. Briefly discuss role of monocytes in generation of atheroma. (supp:2010) User Monocytes transform into macrophages and avidly engulf lipoproteins, including oxidized LDL. Monocytes recruitment & differentiation into macrophages & ultimately into foam cells is theoretically protective, since these cells remove potentially harmful lipid particles. However progressive accumulation of oxidized LDL drive lesion progression. Thus macrophages activation results in cytokines production (TNF) & reactive oxygen species thus aggravating LDL oxidation. Q.NO.3. A 65 year old dictator of banana republic, who was an alcoholic and fond of red meat, suffered a short episode of unexplained chest pain after he was forced to resign and died before he could reach the hospital. At autopsy the pathologist found thickened walls of many arteries including the coronary arteries with luminal narrowing. The lesion consisted of raised plaques having a soft center with a fibrous cap: (annual 2008) a) what is the process known as and what other arteries it most commonly involves? b) what are the principle components of these plaques? a. atherosclerosis abdominal aorta is involved much greater degree than thoracic aorta..in this order: . lower abdominal aorta . coronary arteries . popliteal arteries . internal carotid artery . vessels of circle of villous b. 1. cells including SMCs, macrophages, & T cells 2. ECM including collagen, elastic fibers, & proteoglycans 3. intracellular and extracellular lipid Q.NO.4.A 58 year old principal of a medical college is diagnosed with acute myocardial infarction after he collapsed on his office desk. He is known hypertensive and smoke 10 to 12 cigarettes per day and drinks socially. Serum cholesterol level is elevated and he confesses being fond of dessert and mithai. His BMI and waist circumference are stable angina. (annual 2007) a. identify the major risk factors for atherosclerosis in this patient and categorize them as consecutive (non modifiable) and potentially controllable.

Edited by Foxit Reader Copyright(C) by Foxit Corporation,2005-2009 KHIZZER KHAN For Evaluation Only.

b. also list the minor risk factors for atherosclerosis in this patient. a. non modifiable: 1. increasing age 2. male gender 3. family history increased for his height. His father died of myocardial infarction and his mother has Potentially controllable: 1. hypertension 2. hyperlipidemia 3. cigarette smoking b. minor: 1. obesity 2. high carbohydrate intake (mithai) 3. stress (office work) 4. physical inactivity ( office work) Q.NO.5. Write down the role of hyperlipidemia in atherosclerosis. (annual 2006) chronic hperlipidemia particularly hypercholesterolemia, can directly impair EC function by increasing local production of reactive oxygen species. Among these effects oxygen free radicals accelerate nitric oxide decay, damping its vasodilator activity and there by increasing local shear stress. with chronic hyperlipidemia, lipoproteins accumulate with in the intima. These lipids are oxides to through the action of oxygen free radicals locally generated by macrophages or ECs. Oxidized LDL is ingested by macrophages through a scavenger receptors, distinct from the LDL receptors resulting in foam cell formation. in addition oxidized LDL stimulate the release of growth factors, cytokines and chemokines by ECs and macrophages that increase monocytes recruitment into lesion. Finally oxidized cytotoxic to the ECs & SMCs & can induce EC dysfunction. the importance of oxidized LDL in atherogenesis is suggested by its accumulation with in macrophages at all stages of accumulation. Moreover, antioxidant therapy (-carotenes & vit E) protects against atherosclerosis Q.NO.6. Define atherosclerosis. Describe the morphological changes in this disease. (annual 2005). atherosclerosis is characterized by initial lesion called atheromas, that protrude into vascular lamina. An atheromatous plaque consists of a raised lesion with a soft yellow, grumous core of lipid (mainly cholesterol and cholesterol esters) covered by a firm, white fibrous cap. Besides obstructing blood flow, atherosclerotic plaque weaken the underlying media & then can themselves rupture, causing acute catastrophic vessel thrombosis. Morphology: atheromatous plaques grossly appear white to yellow, vary 0.3cm to 1.5 cm in diameter. abdominal aorta is involved much greater degree than thoracic aorta..in this order: . lower abdominal aorta . coronary arteries . popliteal arteries . internal carotid artery . vessels of circle of villous principals cells of plaque

Edited by Foxit Reader KHIZZER KHAN Copyright(C) by Foxit Corporation,2005-2009 For Evaluation Only.

1. cells including SMCs, macrophages, & T cells 2. ECM including collagen, elastic fibers, & proteoglycans 3. intracellular and extracellular lipid fibrous cap necrotic core neovascularization calcification rupture, ulceration or erosion hemorrhage atheroembolism aneurysm formation Q.NO.7. a. Define atherosclerosis. (annual 2004) b. enlist risk factors leading to atherosclerosis. c. enlist complications of atherosclerosis along with pathogenesis of each. a. atherosclerosis is characterized by initial lesion called atheromas, that protrude into vascular lamina. An atheromatous plaque consists of a raised lesion with a soft yellow, grumous core of lipid (mainly cholesterol and cholesterol esters) covered by a firm, white fibrous cap. Besides obstructing blood flow, atherosclerotic plaque weaken the underlying media & then can themselves rupture, causing acute catastrophic vessel thrombosis. b. Major risks Lesser risks Nonmodifiable Obesity Increasing age Physical inactivity Male gender Stress (type A personality) Family history Postmenopausal estrogen deficiency Genetic abnormality High carbohydrates intake Potentially controllable Lipoprotein (a) Hyperlipidemia Hardened trans unsaturated fat intake Hypertension Chlamydia pneumoniae infection Cigarette smoking Diabetes C- reactive proteins c. complications: Vessels weakness: (abdominal aortic aneurysm) Vessels thrombosis: 1. acute MI (coronary artery) 2. stroke (internal carotid & middle cerebral artery) 3. small bowel infarction ( superior mesenteric artery) Hypertension: renal artery atherosclerosis may activate the rennin angiotensin system. Peripheral vascular disease: 1. increased risk of gangrene 2. pain in buttock and when walking (claudication) Cerebral atrophy: atherosclerosis involving circle of willis or internal carotid artery. Q.NO.8. a. Define & classify aneurysms. (Supp: 2004) b. describe aetiology and morphology of dissecting aneurysm. c. give its clinical course. a. localized abnormal dilation of the blood vessels or heart.

Edited by Foxit Reader KHIZZER KHAN Copyright(C) by Foxit Corporation,2005-2009 For Evaluation Only.

abdominal aortic aneurysm mycotic aneurysm berry aneurysm of cerebral artery syphilitic aneurysm aortic dissection b. pathogenesis: 1. cystic medial degeneration ( CMD ): a. elastic tissue fragmentation b. matrix material collects in the area of fragmentation in the tunica media 2. Risk factors for CMD: a. increase in wall stress: hypertension, pregnancy, coarctation b. defect in connective tissue: marfan syndrome, Ehlers-Danlos syndrome, Vitamin C deficiency, copper metabolic defects. c. skeletal abnormalities: elongated axial bones d. ocular findings: lens subluxation 3. Intimal tear: a. due to hypertension or underlying structural weakness in the media b. usually occur with in 10cm of aortic valve. c. blood dissects under aterial pressure through area of weakness d. blood dissect proximally and distally morphology: the intimal tear marking the point of origin of the dissection is found in the ascending aorta, usually with in 10cm of aortic wall. Such tears are usually transverse or oblique & 1-5 cm in length with sharp, jagged edge. dissection can extend along the aorta retrograde toward the heart as well as distally Some times all the way into the iliac and femoral A. dissecting hematoma spreads along the laminar plane of aorta b/w middle & outer 3rd. it often rupture out through the adventitia causing massive hemorrhage. some times hematoma reenters the lumen of the aorta, producing second distal intimal tear & new vascular channel with in the media of aortic wall double barreled aorta cystic medial degeneration (CMD) Inflammation is absent Clinical course: Type A dissection:(proximal lesion) involving either ascending aorta or both ascending & descending aorta (type 1 & 11 of the DeBakey classification) Type B: (distal lesion): not involving ascending part & usually beginning distal to the the subclavian artery called type B or DeBakey 111). sudden onset of excruciating pain in the chest radiating to back b/w scapulae rupture: usually into pericardial sac, pleural cavity or peritoneal cavity cardiac temponade aortic insufficiency myocardial infarction extension of the dissection into the great aretery of the neck loss of upper extremity pulse ( due to compression of subclavian) aortic valve regurgitation Q.NO.9. Discuss the morphology and aetiology of atherosclerosis. (2003).

Edited by Foxit Reader Copyright(C) by Foxit Corporation,2005-2009 KHIZZER KHAN For Evaluation Only.

Pathogenesis: endothelial cell damage of muscular and elastic arteries causes of endothelial cell injury ( hypertension, smoking, tobacco & LDL ) cell response to the endothelial injury 1. Macrophages & platlets adhere to damaged endothelium 2. release cytokines causes hyperplasia of medial smooth muscle cells 3. smooth muscle cells migrate to the tunica media 4. cholesterol enters smooth muscle cells & macrophages called foam cell. 5. smooth muscle cells release cytokines that produce extracellular metrix. 6. matrix components include collagen & proteoglycans & elastin. development of fibrous cap 1. component of fibrous cap: ( smooth muscle, foam cells, inflammatory cells, ECM 2. fibrous cap overlies a necrotic centre 3. disrupted plaques may extrude underlying necrotic material leading to thrombosis. fibrous plaque becomes dystrophically calcified & ulcerated Q.NO.10. a. Define atherosclerosis. b. discuss very briefly role of of HDL-cholesterol and LDL cholesterol in atherogenesis. c. discuss briefly the complications of atherosclerosis. (supp: 2003) b. the major component of the serum cholesterol associated increased risk is LDL (bad cholesterol). It delivers cholesterol the peripheral tissues In contrast HDL (good cholesterol) mobilize cholesterol from developing & existing atheromas & transports it to the liver for excretion in the bile. So higher level of HDL correlate with reduced risk. Q.NO.11. a) define atherosclerosis. (SZMC sendup 2011) b) outline the risk factor involved. See Q.NO.7 Q.NO.12. Enumerate causes of various types of hypertension. 1. essential hypertension: (90-95%) 2. secondary hypertension 3. renal: acute glumerulonephritis chronic renal disease polycystic disease renal artery stenosis renal vasculitis rennin producing tumors 4. endocrinal: adrenocortical hyperfunctioning ( cushing syndrome) pheochromocytoma acromegaly hypothyroidism (myxedema) hyperthyroidism (thyrotoxicosis) pregnancy induced 5. cardiovascular: coarctation of aorta polyarteritis nodosa increased intravascular volume increased cardiac out put

Edited by Foxit Reader KHIZZER KHAN Copyright(C) by Foxit Corporation,2005-2009 For Evaluation Only.

rigidity of the aorta 6. neurologic hypertension: psychogenic increased intracranial hypertension sleep apnea acute stress (including surgery)

Edited by Foxit Reader KHIZZER KHAN Copyright(C) by Foxit Corporation,2005-2009 For Evaluation Only.

Chapter 12 The Heart Q.NO.1.A 50 years old male presented in emergency room with sever, crushing substernal chest pain radiating to the neck and jaw with discomfort in the epigastrium. The pain had been there for the last 20 minutes and was not significantly relieved by nitroglycerin or rest. On investigation he was found to be a heavy smoker for last 20 years with strong family history of hypercholesterolemia. Elecrocardiograhic abnormalities such as Q waves & ST segment abnormalities & T waves inversion are noted. His blood chemistry is ordered. The patient went into cardiogenic shock and could not come out of it. (annual 2011). a. what are morphologic changes seen in myocardial infarction? b. give the laboratory evaluation a patient with MI. a. reversible injury: Time Gross features Light microscopy Electron microscopy 0-1/2 hr None None Relaxation of myofibrils: glycogen loss. Mitochondrial swelling Irreversible injury: Time Gross Microscopy -4 hr None Weaviness of fiber at border 4-12 hr Occasionally dark mottling Beginning coagulation necrosis; edema, hemorrhage 12-24 hr Dark mottling Necrosis continues, pyknosis of nuclei, myocyte hypereosinophila, marginal contraction band necrosis 1-3 days Mottling with yellow tan Coagulation necrosis, with loss of nuclei and infarct centre striations, interstitial infiltrate of neutrophils 3-7 days Hyperemic border, central Beginning disintegration of dead myofibers yellow tan softening with dying neutrophils. Early phagocytosis py macrophages 7-10 days Maximally yellow tan and Well developed phagocytosis of dead cells soft with red tan margins Granulation tissue formation 10-14 Red gray depressed infract Well established granulation tissue with days border collagen deposition 2-8 wk Gray white scar Increase collagen deposition >2 mo Scarring complete Dense collagen scar b. cardiac troponin T & I: detectable after 2 to 4 hours and peak at 48 hours. Their levels remain elevated for 7 to 10 days. CK-MB: rise with in 2-4 hours, peak at 24-48 hrs & return to normal with in 72hrs. detection of myoglobin detection of lactate dehydrogenase

Q.NO.2.A 70 years old male chronic smoker was brought to the emergency department with a history of acute chest pain with sweating developing into a state of shock. He

Edited by Foxit Reader Copyright(C) by Foxit Corporation,2005-2009 KHIZZER KHAN For Evaluation Only.

expired on arrival in the hospital. Medical autopsy was carried out where pathologist observed dark mottling of anterior wall of left ventricle, anterior part of septum and apex. The cause of death was declared as Myocardial infarction. (annual 2009) a. which coronary artery was critically stenosed in this patient? b. what must have been the duration of survival after the MI and what would be the microscopic picture of this infracted area? c. enumerate complications of myocardial infarction. a. lift coronary artery b. 12-24 hours . For microscopic picture see above c. contractile dysfunction arrhythmias myocardial rupture pericarditis infarct expansion mural thrombus ventricular aneurysm papillary muscle dysfunction progressive late heart failure Q.NO.3. While performing an autopsy on a prison inmate, the pathologist notices a gray white scar in the left ventricle wall. The microscopic section from the lesion shows collagen deposition with out granulation tissue. (Supp:2007) a. if the lesion represents a MI, what is minimum length of time for which the patient must survived after the infarction? b. give the gross and microscopic features of irreversible myocardial injury at 8 hours and 10 days of age. a. 2-8 weeks b. see Q.NO.1 Q.NO.4. Enumerate and briefly describe the complications of MI.(annual 2005). See Q.NO.2 Q.NO.5. a. give an outline of pathogenesis of rheumatic heart disease. (annual 2003) b. enumerate complications & account lab diagnosis of rheumatic heart disease. c. what are heart failure cells? a. it is hypersensitivity reaction induced by host antibodies elicited by group A streptococci. M protein of certain streptococcal strain induce host antibodies that cross react with glycoprotein antigen in the heart, joint & other tissues. This explain the symptoms onset after the pharyngitis.

b.Complications: thromboembolism infective carditis arrhythmias cardiac hypertrophy fibrosis

Edited by Foxit Reader KHIZZER KHAN Copyright(C) by Foxit Corporation,2005-2009 For Evaluation Only.

Diagnosis: serology: culturing the bacteria Jones criteria: carditis migratory polyarthritis of of the large joint subcutaneous nodules erythema marginatum of the skin sydenham chorea c. heart failure cells: heart failure cells or siderophages are hemosiderin containing macrophages in the alveoli of the lungs. The main causes are left heart failure & chronic pulmonary edema. Q.NO.6. A 65 years old male presented with left sided chest pain radiating to left arm and jaw. He was taken to hospital where ECG showed ST elevation. (class test szmc.ryk). a. what is your diagnosis? b. what morphological changes take place in this disease with time? a. Myocardial infarction b. see Q.NO.1 Q.NO.7. A man age 44 seeks advice from a cardiologist. The patient developed peripheral edema in ankles & feet. He is chronic smoker & is diagnosed as a case of corpulomonale with right sided heart failure. (SZMC sendup 2011) a) give schematic representation of sequence of events in heart failure. b) describe morphologic feature of right sided heart failure. a.

b. nutmeg liver centrilobular necrosis cardiac cirrhosis congestive splenomegaly pleural effusion pericardial effusion anasarca (generalized massive edema) Q.NO.8. Enlist the causes of Sudden Cardiac Death? congenital coronary arterial abnormalities aortic valve stenosis mitral valve prolapse myocarditis or sarcoidosis dilated or hypertrophied cardiomyopathy pulmonary hypertension congenital acquired abnormality of cardic conducting system

Edited by Foxit Reader Copyright(C) by Foxit Corporation,2005-2009 KHIZZER KHAN For Evaluation Only.

Chapter 13,14

The Hematopoietic & Lymphoid Systems


Q.NO.1. With acute blood loss, the immediate threat to the patient is hypovalemic shock rather than the anemia. If the patient survives, hemodilution begins at once and achieves its full effect with in 2 or 3 days, unmasking the extent of red cell loss. The anemia is normocytic and normochromic. Recovery from blood loss anemia is enhanced by a rise in the erythropoietin level, which stimulates red blood cell production with in several days. the onset of the marrow response is marked by reticulocytosis. Keeping in view this information give: (annual 2011) a. adult reference ranges for red blood cells. b. morphological and diagnostic criteria of iron deficiency anemia. a. male: 5-5.5M/cub.m female: 4-4.5M/cub.m b. morphology: RBCs are microcytic hypochromic MCV MCHC platelets count erythropoietin bone marrow cellularity Diagnostic criteria: anemia hypochromic& microcytic RBCs indices low serum ferritin low serum iron level low transferrin saturation increase iron binding capacity better response to iron therapy Q.NO.2.A 30 years old woman has a long standing history of menorrhagia. (annual 2010 a. what etiological type of anemia would you expect in this patient? b. what would her peripheral smear and bone marrow aspirate show? c. list three serological tests that you would consider in the patient. a. iron deficiency anemia b. peripheral smear microcytic hypochromic MCV MCHC platelets erythropoietin Bone marrow: marrow cellularity is slightly increased extramedullary hematopoiesis is uncommon c. test: 1. total iron binding capacity (TIBC) 2. CBC 3. Hb & haematocrit 4. peripheral blood smear

Edited by Foxit Reader KHIZZER KHAN Copyright(C) by Foxit Corporation,2005-2009 For Evaluation Only.

Q.NO.3. Peripheral blood film of a 23 years old female with history of jaundice and sever anemia shows the numerous sickle shaped red cells. (annual 2008) a. what type of hemoglobin would the Hb electropherosis show and how is it formed? b. outline the mechanism of sickling of red cells in this patient. a. HbS B. mechanism of sickling: 1. predominantly extravascular hemolysis of sickle cells 2. missense point mutation ( replacement of glutamic acid by valine at 6 position of globin chain) 3. causes of sickling: a.HbS molecules aggregate & polymerize into long needle like fiber. RBCs assumes sickle or boat like shape b. HbS concentration greater than 60% is the most important factor for sickling. c. increase in deoxyhemoglobin increase the risk for sickling acidosis volume depletion hypoxemia Q.NO.4. A 50 years old male patient develops generalized lymphadenopathy with low grade evening pyrexia. Biopsy of lymph nodes shows effacement of architecture with scattered classic read Sternberg (RS) cells in a polymorphous background:supp:2008) a. what is the diagnosis? b. list the five subtypes of this disease with the type of RS cells characteristic of each. a. Hodgkin lymphoma b. nodular sclerosis Hodgkin lymphoma mixed cellularity Hodgkin lymphoma lymphocyte predominance Hodgkin lymphoma lymphocyte rich Hodgkin lymphoma lymphocyte depletion Hodgkin lymphoma Q.NO.5. Illustrate the pathogenesis of anemia, skeletal deformities, and hemochromatosis in beta thalassemia major with the help of a flow chart. (annual 2007)

Edited by Foxit Reader KHIZZER KHAN Copyright(C) by Foxit Corporation,2005-2009 For Evaluation Only.

Q.NO.6. Classify Hodgkin lymphoma. Write morphology of mixed cellularity type of Hodgkin lymphoma. Tabulate the clinical differences b/w Hodgkin and non- Hodgkin lymphomas. (annual 2006) nodular sclerosis Hodgkin lymphoma mixed cellularity Hodgkin lymphoma lymphocyte predominance Hodgkin lymphoma lymphocyte rich Hodgkin lymphoma lymphocyte depletion Hodgkin lymphoma Morphology of mixed cellularity: most common form of Hodgkin lymphoma in patient older than 50, comprises 25% there is male predominance classic RS cells are plentiful with in a distinctive heterogeneous cellular infiltrate which includes lymphocytes, eosinophils, plasma cells and benign histiocytes. compared with other subtype it have disseminated disease &systemic manifestation Hodgkin lymphoma Non-Hodgkin lymphoma 1. more often localized to single axial More frequent involvement of multiple Group nodes ( cervical, mediastinal) peripheral nodes 2. orderly spread by contiguity Noncontiguous spread 3. mesenteric nodes waldyer ring rarely Mesenteric nodes and waldyer ring Involved commonly involved 4. extra-nodal involvement uncommon Extra- nodal involvement common Q.NO.7. Classify Hodgkin lymphoma. Write morphology of mixed cellularity type of Hodgkins disease. (annual 2005) See Q.NO.6 Q.NO.8.a.Classify anaemias on the basis of RBCS morphology and aetiology. b. enlist causes of aplastic anaemia. Give salient features bone barrow smear of aplastic anemia. (annual 2004) 1.microcytic anemias: MCV<80 m3 iron deficiency anemia anemia of chronic disease

Edited by Foxit Reader KHIZZER KHAN Copyright(C) by Foxit Corporation,2005-2009 For Evaluation Only.

thalassemia sideroblastic anemia 2. macrocytic anemia: MCV> 100 m3 folate deficiency vit B12 deficiency 3. normocytic anemia: MCV 80-100m3 a. blood loss & aplastic anemia b. intrinsic RBCs defect: hereditary spherocytosis hereditary ellipocytosis paroxysmal nocturnal sickle cell anemia G6PD deficiency c. extrinsic RBCs defect: immune hemolytic anemia microangiopathic hemolytic anemia malaria a. causes of aplastic anemia: idiopathic: 50-70% drugs: alkylating agent & antimetabolites idiosyncratic reaction to mylotoxic (chloramphenical, sulphonamides) infections: EBV, CMV, parvovirus & heap A,B,C are not culprit physical agents: ionizing radiation ( therapeutic or nuclear accident) miscellaneous: thymoma, paroxysmal nocturnal hemoglobinuria Bone marrow findings: hypocellular bone marrow 90% intertrabecular space being occupied by fat the limited cellurity often consists of only lymphocytes & plasma cells reticulocytopenia pancytopenia erythropoisis is depressed increased fat stasis Q.NO.9. a. enlist causes of generalized lymphadenopathy. (supp:2004) b. tabulate the features of tuberculosis lymphadenopathy vs lymphoma. c. draw and label microscopic picture of mixed cellularity Hodgkin lymphoma. a. Viruses. CMV, EBV Sexual transmitted diseases (STDs) Toxoplasma Hepatitis B Rehumatic arthritis SLE Breast cancer AIDS TB

Edited by Foxit Reader KHIZZER KHAN Copyright(C) by Foxit Corporation,2005-2009 For Evaluation Only.

Q.NO.10. a. classify anaemias. (annual 2003) b. discuss laboratory diagnosis of megaloblastic anemia. c. tabulate differences b/w childhood adult types chronic myeloid leukemia. a. b. lab diagnosis: Hb: CBCs: ( RBCs, WBCs & platelets): (MCV, RDW, MCHC ) Peripheral blood picture: 1. myloid mass will be decreased 2. macrocytosis (oval mycrocyte) 3. anisocytosis 4. poiklocytosis 5. hypersegmentation of neutrophil 6. low platelets & WBCs bone marrow examination: 1. hypercellular fragments 2. megaloblast 3. myloid series shift to lift serum bilirubin level increased raised LDH Decreased hepatoglobin decreased serum vit B12 level schilling test: to rule out the cause of vit B12 deficiency FIGLO test:differentiate either the anemia is due to vitB12 or folic acid c. Q.NO.11. Classify non-Hodgkin lymphoma. b. discuss very briefly Burkitts lymphoma. c. what are Reed Sternberg and lacunar cells? (supp:2003) a. 1. B-cell lymphoma: Burkitts lymphoma diffuse large B-cell lymphoma extranodal marginal zone lymphoma follicular lymphoma small lymphocytic lymphoma 2. T-cell lymphoma Precursor T-cell lymphoblastic leukemia mycosis fungoides & sezary syndrome b. Burrkitts lymphoma: 30b% of children with non Hodgkin in USA. EBV infection Morphology: the tumor cell have oval nucleoli containing two to five prominent nucleoli. high mitotic rate starry sky pattern Clinical features: American type: GIT, para-aortic nodes African type: jaw bone marrow involvement, lukemic phase c. Reed Sternberg cell: 1. neoplastic cell of Hodgkin lymphoma a) in most cases it is a transformed germinal centre B cells b) CD15, CD30 positive 2. classic RS cell: two mirror image nuclei or nuclear lobes, each containing a large (inclusion like) acidophilic nucleolus surrounded by a distinctive Clear zone, together they impart an owl eye appearance.

Edited by Foxit Reader KHIZZER KHAN Copyright(C) by Foxit Corporation,2005-2009 For Evaluation Only.

Nuclear membrane is distinct. lacunar cells: a) pale yellow with multilobed nucleus, containing many small nuclei b) cell lies with in a nuclear space in formalin fixed tissue. c) present in nodular sclerosing types Q.NO.12. a. classify Hodgkin lymphoma according to WHO classification. b. describe morphological appearance of Reed Sternberg cell. c. name two variants of Reed Sternberg cell. (class test szmc.ryk) a. b. 1. neoplastic cell of Hodgkin lymphoma a) in most cases it is a transformed germinal centre B cells b) CD15, CD30 positive 2. classic RS cell: : two mirror image nuclei or nuclear lobes, each containing a large (inclusion like) acidophilic nucleolus surrounded by a distinctive Clear zone, together they impart an owl eye appearance. Nuclear membrane is distinct. c. RS variants: 1. L & H variant: a) large, pale staining, multilobed cell ( popcorn cell) b) present in lymphocytes predominant types 2. lacunar cells: a) pale yellow with multilobed nucleus, containing many small nuclei b) cell lies with in a nuclear space in formalin fixed tissue. c) present in nodular sclerosing types Q.NO.13.a. Describe clinical features of Hodgkin lymphoma. (class test szmc.ryk) b. write down clinical staging of Hodgkin lymphoma in tabulate form. a. constitutional signs: a. fever, unexplained weight loss, night sweats b. puritus c. Pel-Ebstein virus: uncommon variant of fever hematologic findings: a. normocytic anemia b. painless enlargement of single group of lymph nodes main factors determining prognosis : a. clinical stage is more important than the type Hodgkins. b. Majority have lymphadenopathy above the diaphragm( stage 1 & 11) c. Usually involves supraclivicular nodes & anterior mediastinal nodes increase risk for second malignancy : a. acute myelogenoous leukemia or NHL b. complication of treatment with radiation & alkylating agents b. Stage Distribution of disease 1 Involvement of single lymph node region (1) or involvement of single extralymphatic organ or tissue (1E) 11 Involvement of two or more lymph node region on the same side of diaphragm alone (11). Or involvement of limited contiguous extralymphatic organ (11E)

111

Involvement of lymph node region on both side of the diaphragm (111) which may include the spleen (111s) limited contiguous extralymphatic organ (111E) or both (111ES) 1V Multiple or disseminated foci of involvement of one or more extralymphatic organ or tissue with or with out lymphatic involvement Q.NO.14. A young adult was diagnosed as a case of acute lymphoblastic leukemia on bone marrow biopsy. (AIMC sendup) a. briefly describe it morphological findings on bone marrow biopsy. b. give immunophenotype of B and T acute lymphoblastic leukemia. a. increased no. of lymphoblast depressed myelopoiesis depressed erythropoiesis there is increased foci of mitosis known as starry sky b. B ALL: CD10, CD19, CD20 positive T ALL: CD1, CD2, CD5, CD7 positive Q.NO.15. a. Define paroxysmal Nocturnal Hemaglobiuria b. outline the clinical findings of PNH. (SZMC sendup 2011) a. Acquired membrane defect in multipotent myeloid stem cell in which intravascular complement mediated lysis of RBCs, neutrophils & platelets occurs at night time, because respiratory acidosis enhance complement attachement to these cells. b. a. episodic hemoglobinuria (may cause iron deficiency) b. increased incidence of vessel thrombosis (hepatic vein) c. increased risk for developing acute myelogenous leukemia d. it screen sucrose hemolysis test ; confirm----acidified serum test Q.NO.16. a. define hemophilia A b. outline the complications. (SZMC.sendup 2011) a. x linked recessive bleeding disorder caused by reduction in factor VIII activity complications: massive hemorrhage after trauma spontaneous bleeding hemarthrosis ( bleeding in joints ) prolonged PTT Q.NO.18. Give revised FAB classification of AML. Class definition M0 Minimally differentiated AML M1 AML with out maturation M2 AML with maturation M3 Acute promyelocytic leukemia M4 Acute myelomonocytic leukemia M5 Acute monocytic leukemia M6 Acute erythroleukemia M7 Acute megakaryocyte leukemia Q.NO.19. give clinical disorders in which splenomegaly occur 1. massive splenomegaly: (weight more than 1000gm) chronic myeloid leukemia chronic lymphocytic leukemia hairy cell leukemia

Edited by Foxit Reader Copyright(C) by Foxit Corporation,2005-2009 KHIZZER KHAN For Evaluation Only.

lymphoma malaria Gaucher disease primary tumor of spleen 2. moderate spleenomegaly: (500-1000gm) Chronic congestive splenomegaly acute leukemia hereditary spherocytosis thalassemia major autoimmune hemolytic anemia amyliodosis Nieman Pick disease TB 3. Mild splenomegaly: (weight <500gm) acute splenitis acute splenic congestion infectious mononucleosis miscellaneous acute fibrile disorders, septicemia

Edited by Foxit Reader Copyright(C) by Foxit Corporation,2005-2009 KHIZZER KHAN For Evaluation Only.

Chapter 15

The Lung
Q.NO.1.A 32 years old male comes to ER with rapid onset of respiratory insufficiency, cyanosis & sever arterial hypoxemia that is refractory to oxygen therapy & progressive multisystem organ failure. He is diagnosed to have ARDS. (annual 2011) a. what clinical disorders are associated with the development of ARDS. b. give pathogenesis of ARDS. a. Direct lung injury Indirect lung injury Common causes 1. pneumonia Sepsis 2. aspiration of gastric content Sever trauma with sock Uncommon causes Pulmonary contusion Cardiopulmonary bypass Fat embolism Acute pancreatitis Near drowning Drug overdose Inhalational injury Transfusion of blood products Reperfusion injury after lung implantation Uremia b. pathogenesis: a. acute damage to alveolar capillary walls and epithelial cell. b. alveolar macrophages and other cells release cytokines 1. cytokines are chemotactic to neutrophils 2. neutrophils transmigrate into alveoli through pulmonary capillaries 3. capillary damage cause leakage of a protein rich exudate producing hyaline membranes by the action of different mediators. 4. neutrophils damage type 1 and type 2 pneumocytes decrease in surfactant causes atelectasis with intrapulmonary shunting c. late findings 1. repair by type 2 pneumocytes 2. progressive interstitial fibrosis Diagram

Edited by Foxit Reader KHIZZER KHAN Copyright(C) by Foxit Corporation,2005-2009 For Evaluation Only.

Q.NO.2.A 65 years old smoker with history of 30 pack year presents with sever dysponea; on examination he is found to have a barrel chest and prolonged expiration. Spirometry shows expiratory airflows limitation. (annual 2010) a. what are the TWO major clinically important types of this disease? b. what are the TWO major pathogenetic mechanisms involved? Give silent features. a. 1. centriacinar (centrilobular) emphysema 2. panacinar (panlobular) emphysema 3. distal acinar emphysema 4. irregular emphysema b. pathogenesis: 1. smoking: reactive oxygen species (free redicals) inactivation of antiproteases (functional alpha-1 AT deficiency neutrophil elastase increases tissue damage emphysema 2. congenital alpha-1 antitrypsin deficiency increases neutrophil elastase tissue damage emphysema protease-antiprotease imbalance oxidant-antioxidants imbalance Diagram:

Q.NO.3. A 32 years old male diagnosed as a case of community acquired pneumonia & was admitted in pulmonology ward. Upper lobe consolidation was seen on chest X-ray. He developed empyema. ( supplementary 2010) a. describe the pathogenesis of development of this complications. b. if he continues to be non responsive to antibiotics what other complications can arise? Q.NO.4. Respiratory functions tests were performed in a 60 years old chronic smoker. His FEV-1 was reduced. He complained of progressively worsening dysponea. Wheezing was present. His chest X-ray at admission revealed evidenced of pneumothorax & lungs collapse. ( annual 2009) a. what further complications can develop in this patient? b. how did smoking play its role in his lung pathology? a. cor pulmonale

Edited by Foxit Reader Copyright(C) by Foxit Corporation,2005-2009 KHIZZER KHAN For Evaluation Only.

congestive heart failure secondary pulmonary hypertension blebs and bullae (large holes in the lungs respiratory acidosis b. see Q.NO.2 Q.NO.5. A 22 years old medical student from Islamabad suffer from seasonal attacks of episodes of respiratory difficulty with cough productive of copious watery secretions. The episodes last for a few hours at a time. A skin test for paper mulberry pollen is positive. (annual 2008) a. what is the diagnosis? Give the pathologic mechanism underlying this condition? b. list the 3 major groups of chemical mediators which are implicated in this response? a. Atopic asthma pathogenesis: 1. Type 1 hypersensitivity reaction with exposure to extrinsic allergens 2. initial sensitization to an inhaled allergen a. stimulate induction of subtype 2 helper T cell that release IL-4 & IL-5 b. IL-4 stimulates IgE production c. IL-5 stimulates eosinophils 3. inhaled antigen cross link IgE antibodies on mast cells on mucosal surfaces a. release of histamine and other preformed mediators b. functions of mediators stimulate bronchoconstriction, mucous productionm, leukocytes influx 4. late phase: (4-8 hours later) a. eotaxin is produced chemotactic for eosinophils and activates eosinophils b. eosinophils release major basic protein and cationic proteins damage epithelial cells and produce airway obstructions chemical mediators: 1. 1st group (putative): IL C4, D4, E4 & acetylcholine 2. 2nd group: histamines, PGD2, PAF 3. 3rd group: IL-1, IL-6 & TNF Q.NO.6. A 6 year old boy living in Islamabad suffer from attacks of sever dysponea, wheezing and cough each year during the spring season. He remains asymptomatic b/w episodes: (supp:2008) a. what is the diagnosis? b. what is pathological basis for this condition? What are acute reactions and late response in its pathogenesis? a. atopic asthma b. see Q.NO.5 Q.NO.7. Following bone marrow transplantation, a patient develops high grade fever with chills; chest X ray shows pulmonary infiltrates. (annual 2007) a. list three major causes of pulmonary infiltrates in such patients with two examples for each? b. what pulmonary infections are likely to occur in an AIDS patient at CD4+ count> 200 cells/mm3? a. virus: cytomegalovirus

Edited by Foxit Reader KHIZZER KHAN Copyright(C) by Foxit Corporation,2005-2009 For Evaluation Only.

bacteria: gram negative & S. aureus fungi: Cryptococcus, penumocystitis, aspergllus drug reactions malignancy b. > 200 : bacterial + tubercular infections < 200 : pneumocystitis pneumonia <50 : M. avium + Cytomegalovirus Q.NO.8. Enumerate the conditions associated with development Acute respiratory distress syndromes. (annual 2006) See Q.NO.1 Q.NO.9. Write a short note on: Lung abscess (annual 2005) Ans: it is supparative process which is characterized by the necrosis of lung tissue For further detail see Q.NO.16 Q.NO.10. Define emphysema and list its types. Give pathogenesis of emphysema. (Annual 2004) See Q.NO. 2 Q.NO.11. a) Define and classify pneumonia. ( supp: 2004) b) give aetiology, gross, microscopic features and fate of lobar pneumonia. a. classification: 1. community acquired acute pneumonia: strep. Pneumoniae H. influenza Moraxella catarrhalis S. aureus L. Pneumophila enterobacteriaceae 2. Community acquired atypical pneumonia mycoplasma pneumoniae Chlamydia coxiella burnetti influena virus respiratory syncytial virus adenovirus 3. Nosocomial pneumonia: gram negative rods enterobacteriaceae (klebsiella, serratia, E. coli) pseudomonas S. aureus 4. Aspiration pneumonia: Anaerobes oral flora ( bacteroides, provetella) admixed withaerobes 5. chronic pneumonia: nocardia actinomyces Mycobacterium histoplasma coccidioides immitis 6. Necrotizing pneumonia and lung abscess: 1. S. Aureus 2 strep. Pyogenes 3. K. pneumoniae 4. Pseudomonas 5. anerobes ( prevotella, bacteriods,) 6. type 3 pneumococcus 7. pneumonia in the compromised host: cytomegalovirus pneumocystitis jiroveci M. avium intracellulare invasive . aspergilosis invasive candidiasis b. morphology & microscopy: congestion: vascular congestion, scattered neutrophils red hepatization: lung lobe has liver like consistency gray hepatization: the lung is dry, gray, firm because the red cells are lysed. resolution; exudates with in the alveoli enzymatically digested c. fate: resolution

Edited by Foxit Reader KHIZZER KHAN Copyright(C) by Foxit Corporation,2005-2009 For Evaluation Only.

Q.NO.12. a) Classify Lung tumors. b) Define atelectatsis. c) Discuss the pathogenesis of bronchectasis. (Annual 2003). a. 1. squamous cell carcinoma 2. adenocarcinoma 3. small cell carcinoma (oat cell carcinoma) : combined small cell carcinoma 4. large cell carcinoma: large cell neuro-enocrine carcinoma 5. adenosquamous carcinoma 6. carcinoma with pleomorphic, sarcomatoid or sarcomatous a. Spindle cell carcinoma b. Giant cell carcinoma 7. carcinoid tumors: (typical, atypical) 8. carcinoma of salivary gland type 9. unclassified tumor b. loss of lung volume caused by inadequate expansion of air spaces types: 1. Resorption atelectasis 2. Compression atelectasis 3. Contraction atelectasis c. pathogenesis of bronchiectasis: 1. cystic fibrosis 2. infections: T.B, adenovirus, s. aureus, H. influenza 3. bronchial obstructions: proximally located bronchogenic carcinoma occludes lumen. 4. primary ciliary dyskinesia: a. absent dynein arm in cilia. b. dyenin arm contain ATPase for movement of the cilia. 5. allergic bronchopulmonary aspergillosis Q.NO.13. Define and classify the emphysema. (supp: 2003). Q.NO.14. a. give differences b/w pancinar and centriacinar emphysema. b. Define Reid index and give its normal value. (class test szmc.ryk) a. Centriacinar Panacinar 1. most commonly seen in smokers Associated with alpha-1 AA deficiency 2. apical segments involve Lower lobe is effected Distal alveoli are spared Involves uniformly all levels Acquired Genetic or acquired b. A ratio between the thickness of submucosal mucous secreting glands & thickness b/w epithelium & cartilage that cover the bronchi. Calculation: RI= gland/wall Normal value: less than 0.4 Q.NO.15.A 60 years old male presented with productive cough having blood in it, weight loss, chest pain and dysponea. (class test szmc.ryk) a. what is most likely diagnosis? b. enumerate steps for diagnosis in this case. c. classify tumors of involved organ.

Edited by Foxit Reader Copyright(C) by Foxit Corporation,2005-2009 KHIZZER KHAN For Evaluation Only.

a. tuberculosis b. diagnosis c. clinical history chest x ray 3 sputum samples obtained for culturing of AFB. Nucleic acid amplication test (NAAT) differentiate it from non TB Mycobacterium. biopsy FBC & Na electrolyte CT scan (differentiate it from cancr) Q.NO.16. A 25 years old female went to pulmonologist at peak of spring season having sever dysponea and wheeze. Her pulmonary function test showed decreased FEV1, serum IgE levels were elevated and blood revealed absolute eosinophilia. a. what is most likely diagnosis? (sendup RMC) b. give four histological findings of this disorder. c. name four chronic restrictive airway diseases. a. atopic asthma b. histological changes in bronchi: 1. thickening of the basement membrane 2. edema and mixed inflammatory infiltrate 3. Hypertrophy of submucosal glands 4. hypertrophy/ hyperplasia of smooth muscles cells Histological changes in the terminal bronchioles: 1. formation of spiral shaped mucous plugs a. contains shed epithelial cells called curschmann spirals b. pathologic effect of major basic protein and cationic protein 2. crystalline granules in eosinophils coalesce to form Charcot Leyden crystals. 3. patchy loss of epithelial cells, goblet cells metaplasia. 4. thick basement membrane c. 1. fibrosing disease: pneumoconiosis, idiopathic pulmonary fibrosis 2. granulomatous: sracoidosis, hypersensitivity pneumonitis 3. eosinophilic 4. smoking related 5. other: pulmonary alveolar proteinosis Q.NO.17. a. Define pulmonary abscess. (class test szmc.ryk) b. name three causative organisms. c. enumerate its four complications. a. suppurative process characterized by the necrosis of lug tissue called pulmonary absces b. 1. S. Aureus 2 strep. Pyogenes 3. K. pneumoniae 4. Pseudomonas 5. anaerobes ( prevotella, bacteriods,) 6. type 3 pneumococcus c. complications: 1. pneumothoraax 2. empyema

Edited by Foxit Reader KHIZZER KHAN Copyright(C) by Foxit Corporation,2005-2009 For Evaluation Only.

3. bronchopleural fistulas 4. meningitis 5. brain abscess 6. septic embolism Q.NO.18. Respiratory functions tests were performed in a 60 years old chronic smoker. His FEV-1 was reduced. He complained of progressively worsening dysponea. Wheezing was present. His chest X-ray at admission revealed evidenced of pneumothorax & lungs collapse. (SZMC. Sendup 2011) a. what further complications can develop in this patient? b. how did smoking play its role in his lung pathology? a. cor pulmonale congestive heart failure secondary pulmonary hypertension blebs and bullae (large holes in the lungs respiratory acidosis b. see Q.NO.2 Q.NO.19.Give differences b/w small cell lung carcinoma & non-small cell lung carcinoma. Features SCLC NSCLC Histology Scant cytoplasm, small Abundant cytoplasm, hperchromatic nuclei, fine pleomorphic nuclei with chromatin, nucleoli of coarse chromatin pattern indistinct Nucleoli prominent Neuroendocrine markers Usually present Usually absent Epithelial markers Present Present Mucin Absent Present Peptide hormones production ACTH, ADH, calcitonin PTH Tumor suppressor gene abnormalities: RB mutations 90% 20% p53 mutations >90% >50% Dominant oncogene abnormalities KRAS mutations Rare 30% EGFR mutations absent 20% Response to chemotherapy & Often complete response Uncommonly complete radiotherapy but recur invariably response

Edited by Foxit Reader Copyright(C) by Foxit Corporation,2005-2009 For Evaluation Only.

KHIZZER KHAN

Edited by Foxit Reader Copyright(C) by Foxit Corporation,2005-2009 For Evaluation Only.

KHIZZER KHAN

Chapter 20

The Kidney
Q.NO.1. Write short notes on nephrotic syndrome. (Annual 2011) The nephrotic syndrome refers to a clinical complex that includes: massive proteinuria, with daily protein loss in the urine is 3.5 gm or more in adults. hypoalbuminemia, with plasma albumin level less than 3gm/dl. generalized edema, the most obvious clinical manifestation hyperlipidemia & lipiduria Causes of nephrotic syndrome: a. primary glomerular disease: membranous GN minimal change disease focal segmental glomerulosclerosis membranoproliferative GN IgA nephropathy b. systemic diseases with renal manifestations: Diabetes mellitus Amyloidosis systemic lupus erythematous ingestion of drugs ( gold, pencillamine, street heroin) infections: (malaria, syphilis, hepatitis B & HIV) malignancy ( carcinoma, melanoma) Miscellaneous ( be sting allergy, hereditry nephritis Q.NO.2.An old girl presents with generalized edema and laboratory results show proteinuria in excess of 3.5gm per day with hypoalbuminemia. The patient improved dramatically on corticosteroid administration. (annual 2010) a. what is the most likely diagnosis? b. what are the light microscopic and ultra-structural and immunofluorescence findings for this disease? a. minimal change disease: b. light microscopic findings: 1. golmeruli appear normal 2. the cell of the proximal convoluted tubules are often heavily laden with protein droplets and lipids but this is secondary to the tubular reabsorption of the lipoproteins passing through the diseased glomeruli 3. this appearance of PCT is the basis for the older term for this disorder, lipid nephrosis Ultrastructral & immunofluorescence findings: 1. even with the electron microscope, the GBM appears normal. 2. the only obvious abnormality is the uniform and diffuse effacement of the foot processes of podocytes. 3. the cytoplasm of the podocytes appears flattened over the external aspect of the GBM obliterating the network of arcades b/w podocytes and GBM. 4. there is also epithelial cell vacuolization, microvillous formation & focal detachment.

Edited by Foxit Reader KHIZZER KHAN Copyright(C) by Foxit Corporation,2005-2009 For Evaluation Only.

Q.NO.3. A young male presents with nephritic syndrome. He is HCV positive on serology. Renal biopsy was carried out which revealed on light microscopy an accentuation the lobular arrangement of glomerular tuft and double contour of GBM. a. what will be the electron and light microscopic picture of this biopsy? b. describe the clinical course of this disease. (supp:2010) a. acute postinfectious (poststreptocoocal) GN. Light Micropscopic picture: 1. the most characteristic change in the postinfectious GN is a fairly uniform increase Celllularity of the glomerular tuft that effects nearly all glomeruli, hence term diffuse 2. the increased cellularity is caused by both proliferation & swelling of endothelial and Meseangial cells and by nuetrophilic and monocytic infiltrate. 3. there is necrosis of the capillary wall. 4. may be crescents with in the unrinary space in response to sever inflammatory injury. Electron microscopy: 1. shows deposited immune complexes arrayed as sub endothelial, intramemebranous or most often subepithelial humps nested against the GBM. 2. meseangial deposited are also occasionally present Immunofluorescence findings: 1. this study reveals scattered granular deposited of IgG & complement with in the capillary wall b. clinical course: malaise nausea nephritic syndrome oliguria azotemia hypertension gross hematuria smoky brown urine serum antistreptolysin O antibodies elevated end stage renal disease Q.NO.4.A school going 7 years old boy complains of few weeks onset of gradual lethargy, fatigue & pass of small volume of dark colored urine following an episode of upper respiratory tract infection.On examination mild hypertension is present.(anual2009) a) what will be the characteristic findings of urine examination in this patient? b) describe the renal biopsy findings. a.

Q.NO.5.A 62 years old female from a remote town in South Punjab presents with generalized body edema. Labs show hypoalbuminomia and microalbuminuria. Fasting blood sugar levels are more than 200mg/dl on two consecutive days. The patient gives

Edited by Foxit Reader Copyright(C) by Foxit Corporation,2005-2009 KHIZZER KHAN For Evaluation Only.

history of fainting episodes for which she had been consulting the local pir. What spectrum of changes would you expect to find on renal biopsy from this patient.(2008) Diabetic nephropathy: 1. Glomerular lesion: 1. thickening of GBM 2. diffuse glomerulosclerosis 3. nodular glomerularsclerosis 4. exudates lesion (capsuar drops) 2. renal vascular lesion: 1. arteriosclerosis 2. hyline arteriosclerosis of both afferent & efferent arterioles. 3. acute & chroni pyelonephritis 4. diffuse mesangial sclerosis 5. necrotizing papillitis 6. tubular lesion: glycogen infiltrate, glycogen nephrosis or armanni ebstein cells Q.NO.6. Sections from transurethral resection of a bladder tumor reveal lamina propria invasion by a papillary urothelial carcinoma. (supp:2008) a. what is pathologic stage T (primary tumor) for this tumor? b. give the American Joint Commission on cancer staging (AGCC) for bladder carcinom c. illustrate the four gross morphologic types of bladder cancer. a. T1 b. Ta Non-invasive, papillary Tis Carcinoma in situ ( noninvasive, flat) T1 Lamina propria invasion T2 Muscularis propria invasion T3a Microscopic extra-vesicle invasion T3b Grossly apparent extra-vesicle invasion T4 Invade adjacent structures c. 1. papilloma papillary carcinoma 2. invasive papillary carcinoma 3. flat non-invasive carcinoma 4. flat invasive carcinoma Q.NO.7. Clinical pyelonephritis is most commonly caused by ascending infection. Give the five major steps in its pathogenesis. (annual 2007) 1. adhesion of bacteria to the mucosal surfaces: a. hematogenous infection: (E.coli, staphylococcus) b. ascending infection: (E.coli, proteus, enterobactor, pseudomonas, klebsiella) 2. instrumentation: catheterization & cystoscopy 3. urinary obstruction: congenital or acquired, (in the stasis of urine bacteria multiply) 4. increased vessicourectal reflex: 5. pregnancy 6. diabetes mellitus 7. preexisting renal lesion 8. patients sex & age: male & advancing age 9. immunosuppression & immunodifiency Q.NO.8. Give the pathogenesis and morphology of acute proliferative post streptococcal glomerulonephritis. (annual 2006)

Edited by Foxit Reader Copyright(C) by Foxit Corporation,2005-2009 For Evaluation Only.

KHIZZER KHAN

Pathogenesis: immune complex deposition on GBM .Typical features of immune complex disease, such as hypocomplementemia & granular deposition of IgG & complement on the GBM. C3 may be deposited on the GBM before IgG. Morphology: see Q.NO.3 Q.NO.9.a. Name causes of painless hematuria. b. describe briefly morphology of transition cell carcinoma. (annual 2006) a. cause of painless hematuria: bening pappiloma urothelial (transitional cell carcinoma) squamous cell carcinoma papilloma papillary carcinoma invasive papillary carcinoma flat non-invasive carcinoma Renal cell carcinoma b. morphology of transition cell carcinoma: Gross and microscopic findings: 1. low grade cancer: (usually papillary and are not invasive) 2. high grade cancer: (papillary or flat and are usually invasive 3. most common sites: lateral or posterior walls at the base of the bladder 4. significance of blood group antigens ( A, B or H ) Q.NO.10. a. Enlist cause of haematuria. a. Upper urinary tract ( kidney, ureter) renal stone glomerulonephritis renal cell carcinoma b. Lower urinary tract ( bladder, urethra, prostate ) infection transitional cell carcinoma benign prostatic hyperplasia c. Drugs associated with hmaturia anticoagulant ( warfarin & heparin ) cyclophosphamide ( hemorrhagic cystitis ) Q.NO.11. Define nephritic syndrome. Enumerate its cause & briefly describe the morphology of membranous glomerulonephritis (MGN). (supp:2003) Nephritic syndrome: this is clinical complex characterized by: hematuria with dysmorphic red cells & red blood cell casts in the urine oliguria & azotemia hyprtension proteinuria Causes: immunologically mediated glomerular injury genetic systemic diseases e.g SLE primary glomerular disease inflammatory reactions Morphology: Under light microscope: diffuse thickening of GBM Under electron microscope: subepithelial deposits separated by small

Edited by Foxit Reader KHIZZER KHAN Copyright(C) by Foxit Corporation,2005-2009 For Evaluation Only.

Spikes of GBM matrix (spike & dome pattern) deposits are incorporated into GBM, giving its double layered appearance with spikes ( rail track pattern ) deposits are later catabolized & disappear leaving behind cavities with in GBM which are filled with progressive deposition of GBM matrix ultimately as disease progress glomeruli become sclerosed and hylinized.There is loss of foot process of podocytes Under Fluorescence microscope: show granular deposits of immunoglobulins & & complement along the GBM. Q.NO.12. a. Classify renal tumors. b. give the morphological features of nephroblastoma (wilms tumor). c. enumerate causes of haematuria. (supp:2003) a. Epithelial tumors of renal parenchyma: 1. adenoma 2. renal cell carcinoma Epithelial tumors of renal pelvis: 1. transitional cell papilloma 2. transitional cell carcinoma 3. squamous cell carcinoma 4. adenocarcinoma of renal pelvis Nephroblastic tumors: 1. nephroblastoma: (Wilms tumor) 2. mesoblastic nephroma 3. multioccular cystic nephroma Non-epithelial tumors: Benign: 1. angiomyolipoma 2. fibroma 3. hemangioma 4. oncocytoma Secondary tumors Un-classified tumors Tumors like lesion: 1. renal blastema 2. renal dysgenesis 3. renal cysts classification of renal cell carcinomas: 1. clear cell carcinoma 2. papillary renal cell carcinoma 3. chromophobe renal cell carcinoma 4. wilms tumor (nephroblastoma) b. gross: unilateral palpable mass in a child with hypertension. Hyperetension due to rennin. large necrotic gray tan tumor dereived from mesonephric mesoderm lungs are the most common site of the metastasis Microscopic: contain aborative glomeruli and tubules, primitive blastemal cells &rhabdomyoblasts shows nests & sheets of primitive blastema with intervening mesenchyme. foci of muscles, bones, cartilage containing cholesterol crystals and lipid macrophages may be seen. degree of anaplasia in the stromal component correlate with prognosis.

Edited by Foxit Reader Copyright(C) by Foxit Corporation,2005-2009 KHIZZER KHAN For Evaluation Only.

Q.NO.13. write down the cause of tubuointerstitial nephritis. infections: ( acute bacterial pyelonephritis, chronic pyelonephritis) toxins: (drugs, analgesic, heavy metals) metabolic diseases (urate nephropathy, acute phosphate nephropathy) physical factors ( chronic urinary tract obstruction) neoplasms (multiple myeloma) immunologic reactions (transplant rejection, sjogren syndrome, sarcoidosis) vascular disease Q.NO.14. Give classification of cystic diseases of the kidney. multicystic renal dysplasia polycystic kidney disease: a. autosomal dominant (adult) polycytic disease b. autosomal recessive (childhood) polycytic disease medullary cystic disease: a. medullary sponge kidney b. nephronothiasis acquired ( dialysis associated) cystic disease localized ( simple) renal cyst renal cyst in hereditary malformation syndrome glumerulocystic disease extraparenchymal renal cysts Q.NO.15. Define obstructive uropathy or hydronephrosis. Enumerate its causes. Hydronephrosis refers to dilation of the renal pelvis & calyces, with accompanying atrophy of the parenchyma caused by obstruction to the outflow of urine Causes: congenital anomalies ( uretharal stricture, meatal stenosis, bladder neck obstruction) urinary calculi benign prostatic hypertrophy (BPH) tumors: (carcinoma of prostate, bladder tumors) inflammation: (prostatitis, uretritis) sloughed papillae or blood clots pregnancy uterine prolapse & cystocele functional disorders: (neurogenic) Q.NO.16. Give main types urolithiasis with prevalence. 1. calcium oxalate & phosphate: 70% 2. magnesium ammonium phosphate: 15-20% 3. uric acid: 5-10% 4. cystine: 1-2%

Edited by Foxit Reader Copyright(C) by Foxit Corporation,2005-2009 For Evaluation Only.

KHIZZER KHAN

Chapter 17

The Oral Cavity & GIT

Q.NO.1. A 60 years old female comes for consultation in your clinic with history of local pain at the base of tongue and difficulty in chewing. Previously she has been asymptomatic that recently she has noticed ulceration at mentioned site. She has suspected to have a neoplastic lesion. (annual 2011). a. what is the risk factors that make her a candidate for a oral cancer? b. give the morphological details of squamous cell carcinoma? a. Factors Comments 1. leukoplaki, 3-25% in leukoplakia, more than 50% in erythroplakia erthyroplakia 2. tobacco use Best established influence, particularly pipe smoking 3. Human papilloma 30-50% chances virus type 16 & 18 4. alcohol abuse Weaker influence than tobacco use 5. protracted irritation Weakly associated b. morphology: 1.location: 1. vermilion border of the lateral margins of the lower lip 2. floor of the mouth 3. lateral borders of the mobile tongue 2. early lesions appear as pearly white to gray circumscribed thickening of the mucosa . . . closely resembling leukoplakia patches. 3. late lesions may grow in a exophytic fashion to produce readily visible and palpable Nodular and fungating lesions. 4. eventually the lesions may assume an endophytic, invasive pattern with central Necrosis to produce cancerous ulcer. Microscopy: 1. moderately to will differentiated keratinizing tumors. 2. epithelial atypia 3. dysplasia 4. carcinoma in situ in the margin. 5. metastasis rarely with lip cancers Q.NO.2. A 50 years old male gives history of epigastric discomfort and weight loss for the last two months. He has had no symptoms before this; endoscopy biopsy was planned and sent for histopathology. The findings are given in diagram. (annual 2011)

Edited by Foxit Reader Copyright(C) by Foxit Corporation,2005-2009 For Evaluation Only. KHIZZER KHAN

a. what is the diagnosis? b. give three microscopic growth patterns of this neoplasm, which may be evident at both the early and advanced age. c. give risk factors for gastric carcinoma. a. Gastric carcinoma b. 1. exophytic 2. flat or depressed 3. excavated c. Intestinal type adenocarcinoma: chronic gastritis with intestinal metaplasia infection with helicobacter pylori nitrites derived from nitrates present in food may undergo nitrosation to form Nitrosamine and nitrosamides diets containing foods that may generate nitrites (smoked foods & vegetables) decreased intake of fresh vegetables and fruits (antioxidants) partial gastrectomy pernicious anemia Diffused carcinoma: risk factors undefined, except for a rare inherited mutations of Ecadherin infection with H. pylori and chronic gastritis often absent Q.NO.3.A 30 years old female has recurrent episodes of bloody diarrhea with long symptom free intervals. Sigmoidoscopy shows proctocolitis with continuous involvement of mucosa and extending to the splenic flexure. (Annual 2010) a. what is your diagnosis? b. list the gross and microscopic findings you expect to find in a colonic resection from this patient. a. ulcerative colitis b. Gross: involves rectum & sigmoid & may involve the entire colon presentation with an even higher proximal extension (pancolitis) occurs much less frequently. Colonic involvement is continuous from the distal colon so, that skip lesions are not encountered inflammatory destruction of the mucosa, with hyperemia, edema &granularity extensive & broad-based ulceration of the mucosa in the distal colon or throughout the its length.

Edited by Foxit Reader Copyright(C) by Foxit Corporation,2005-2009 KHIZZER KHAN For Evaluation Only.

isolated islands of regenerating mucosa bulge upward to create pseudopolyps. colon progressively swells & becomes gangrenous ( toxic megacolon ) chronic mucosal damage Microscopic: a diffuse, predominantly mononuclear inflammatory infiltrate in the lamina propria is almost universally present. nuetrophils infiltrate of epithelial layers may produce collections of these cells in crypt lumina (crypt abscess) granulation tissue fills in the ulcer crater, followed by regeneration. submucosal fibrosis & mucosal architectural dissarry & atrophy remain as residua of healed disease. Q.NO.4.A 65 years old man presents with anemia and stool for occult blood is positive. A 3.5 cm sessile cauliflower like mass is noted in the rectum on sigmoidoscopy. The remaining mucosa appears normal. (annual 2010) a. list the THREE types of adenomas you will consider in the differential diagnosis. b. what factors determine the risk of malignancy in similar case? a. villous adenomas D/D: 1. Tubular adenomas 2. Tubulovillous adenomas 3. sessile serrated adenomas b. three factors determine the risk of malignancy: 1. polyp size (maximum diameter is the chief determinant of the risk of malignancy) 2. histological architecture (does not provide substantive independent information) 3. severity of epithelial dysplasia cancer is rare in tubular adenoma smaller than 1cm in diameter. likelihood of cancer is high in sessile villous adenomas larger than 4cm in diameter sever dysplasia, when present it is often found in villous area. Q.NO.5.A 70 year old male gave history of bleeding per rectum and tenesmus for the past three months followed by sudden development of features of intestinal obstruction. Emergency

Edited by Foxit Reader Copyright(C) by Foxit Corporation,2005-2009 KHIZZER KHAN For Evaluation Only.

laparotomy revealed an obstructing tumor in the rectum which histologicaly turned out to be an adenocarcinoma. (supp: 2010) a. what prognostic factors will determined the survival of this patient? b. describe the role of APC gene in colorectal carcinoma.

Q.NO.6. In a village in Gujranwala an outbreak occurred with school children getting fever, abdominal pain and diarrhea. The labs results were; TLC:3,500; DLC lymphocytes 75%, smear negative for malarial parasites. Widal test was positive during 2nd week of illness. Cases started presenting with intestinal perforation as well as an acute abdominal emergency. (supp:2010). a. what will be the gross appearance of intestine in such a patients? Compare these features with that of tuberclosis intestine. b. what is pathogenesis of intestinal pathology in these children? a. typhoid fever

Q.NO.7.A year old male presents with a history frequqnt attacks of cutanous flushes/ cyanosis, diarrhea, cough, wheezing and enlarged nodular liver. On CT scan a tumor was detected in the terminal ileum with hepatic metastasis. (annual 2009) a. what is tumor most possibly? b. give its morphology. c. explain generation of symptomatology by this tumor. Q.NO.8.A 25 years old male gave few months history of dysponea. On gastroscopy gastric ulcer was seen. The gastric

Edited by Foxit Reader Copyright(C) by Foxit Corporation,2005-2009 KHIZZER KHAN For Evaluation Only.

biopsy revealed heavy colonization by small curved spiral organisms on the surface of mucosa. (annual 2009) a. name the causative organism. b. what is the role of this micro organism in producing this gastric pathology? a. H. pylori b. induces intense inflammatory and immune response. Increase production of pro-inflammatory cytokines such as IL-1, IL-6, TNF and IL-8. IL8 recruits & activates neutrophils. Vacuolating toxins called VacA causes epithelial injury regulated by cytotoxin Associated gene (CagA). This cluster of 29 gene encode proinflammatory proteins. secretes urease that breakdown the urea to form toxic compounds such as Ammonium chloride and monchloramine. bacterial proteases and phospholipase break down the glycoprotein lipid complexes. enhances the gastric secretions and reduces the bicarbonate so pH metaplasia. Its proteins are immunogenic so evoke immune response and activates T & B cells. Q.NO.9.An endoscopic gastric biopsy from a 65 years old man presenting with epigastric discomfort reveals a moderately differentiated glandular tumors not invading beyond submucosa. Adjacent mucosa shows ulceration and presence of H.pylori. subsequent gastrectomy shows a 1 cm flat lesion with perigastric lymph nodes metastasis.(2008) a. what morphological features has the greatest impact on clinical outcome? b. list important environmental, host and genetic risk factors for gastric carcinoma. a. lymph nodes metastasis b. see above Q.NO.10.A colectomy specimen from a 35 years old female with history of intermittent attacks of mild diarrhea with fever and abdominal pain, shows skip lesion in the form of sharply demarcated areas of ulceration and fissuring, also involving the distal ileum. What features would you expect to find on microscopic examination of diseased segments. (annual 2008). Crohn disease: Microscopy:

Edited by Foxit Reader KHIZZER KHAN Copyright(C) by Foxit Corporation,2005-2009 For Evaluation Only.

inflammation with neutrophilic infiltration into epithelial layer and accumulation With in crypt to form crypt abscess ulceration, which is usual outcome of active disease chronic mucosal damage in the form of architectural distortion, atrophy & metaplasia granulomas Q.NO.11. a. How will you differentiate diffuse type of gastric carcinoma from the . . . . intestinal type on histology? b. define early gastric carcinoma and Advanced gastric carcinoma. List four host factors associated with increased incidence of gastric carcinoma (supp:2008) a. diffuse variant: is arised de novo from native gastric mucous cells, is not associated with chronic gastritis, tend to poorly differentiated & less common type intestinal variant: is arised from the gastric mucous cells that have undergone intestinal metaplasia in the setting of chronic gastritis. Better differentiated & more common type b. early gastric carcinoma: defined as lesion confined to the mucosa & submucosa regardless of presence & absence of perigastric lymph nodes metastasis. Advanced gastric carcinoma: is a neoplasm that has extended below the submucosa into the muscular wall & metastases more widely. Q.NO.12. A 55 years old male patient is found to have a 3cm polyp in the right colon while he is being evaluated for anemia: (supp:2008) a. list three non-neoplastoic and three neoplastic types of colorectal polyps. b. give major histologic features of the three types of colorectal adenoma. a. non-neoplastic polyps: 1. hyperplastic polyps 2. hamartomatous polyps 3. juvenile polyps 4. peurtz- jeghers polyps 5. inflammatory polyps 6. lymphoid polyps neoplastic epithelial lesion: 1. benign polyps (adenomas) 2. malignant lesions. (adenocarcinoma, squamous cell carcinoma of the anus)

Edited by Foxit Reader Copyright(C) by Foxit Corporation,2005-2009 For Evaluation Only.

KHIZZER KHAN

b. tubular adenoma: stalk is covered by normal colonic mucosa but the head is composed of neoplastic epithelium. forming branching glands lined by tall, hyperchromatic cells thses cells may or may not show mucin secretion small foci of villous architecture dysplasia or cytoplasmic atypia Villous adenomas: frondlike villiform extensions of mucosa covered by dysplastic, piled up columnar epithelium. all degree of dysplasia encountered invasive carcinoma Tubulovillous adenomas: composed of broad mix of tubular & villous areas they have intermediate degree of dysplasia risk of harboring intramucosal or invasive carcinoma Q.NO.13.A 45 year old male patient is diagnosed with H pylori associated gastritis. List four major diseases associated with infection with the type of association for each. b. what constitutes active inflammation in chronic gastritis? (annual 2007)

Q.NO.14.A 28 year old female has recurrent attacks of bloody mucoid diarrhea with abdominal cramps, which are relieved on defecation. Each episodes lasts for 4-5 days followed by the symptom free periods of 3-4 months. Colonscopy shows pancolitis extending from rectum to the splenic flexure. (annual 2007) a. what is the diagnosis? b. what features will a colonoscopic biopsy show? c. what is the role of intestinal flora in the pathogenesis? a. ulcerative colitis b. c. Q.NO.15. Define peptic ulcer. Explain briefly mechanism of gastric ulcer. (annual 2006) Definition: the chronic, most often solitary, lesions that occurs in any portion of the GIT exposed to aggressive action of acidic peptic juices.

Edited by Foxit Reader Copyright(C) by Foxit Corporation,2005-2009 For Evaluation Only.

KHIZZER KHAN

Pathogenesis: increased damage or impaired defenses by: H. pylori infection NSAID Aspirin Cigarettes Alcohol Gastric hyperacidity Deodenal gastric reflex So ischemia, shock, delayed gastric emptying and host factors cause ulceration. Q.NO.16. Give gross and histological features of basal cell carcinoma. (annual 2006)

Q.NO.17. a. enumerate the ulcero-inflammatory lesions of small intestine and large Large intestine. b. describe morphology of crohns disease. (annual 2005) ans: a. 1. crohn disease 2. ulcerative colitis b. morphology: gross: sharply delimited and typically transmural involvement of the bowel by an inflammatory process with mucosal damage the presence of non-caseating granulomas in 40-60% fissuring with formation of fistula serosa become granular and dull gray and often mesenteric fat wraps around the Bowel surface (creeping fat) intestinal wall is rubbery & thick the result of edema, inflammation, fibrosis skip lesion string sign seen by radiology Microscopy: inflammation with neutrophilic infiltration into epithelial layer and accumulation With in crypt to form crypt abscess ulceration, which is usual outcome of active disease chronic mucosal damage in the form of architectural distortion, atrophy &

Edited by Foxit Reader Copyright(C) by Foxit Corporation,2005-2009 For Evaluation Only.

KHIZZER KHAN

metaplasia granulomas Q.NO.18. give pathogenesis of shistosomiasis (annual 2004).

Q.NO.19. Describe the aetiology, pathogenesis & morphology of peptic ulcer. (supp:2003) Morphology: favored sites are anterior and posterior wall of the first portion of the deudenum a) clean, sharply demarcated & slightly elevated around the edges b) most gastric ulcers are benign c) histology: 1. necrotic debris 2. inflammation with predominance neutrophil 3. granulation tissue 4. fibrosis Q.NO.20. Write short note on: (Class test szmc.ryk) i. Hirschsprung disease ii. tabulate difference b/w ulcerative colitis & crohns disease. i. Distension of colon to greater than 6 or 7 cm in diameter ( megacolon) causes: congenital: mutation in RET genes & RET ligands. During development, the migration of neural crest derived cell along the alimentary tract arrests at some points before reaching the anus. Hence an aganglionic segments is formed that lacks both the Mesissner submucosal & Auerbach myentric plexuses. This causes functional obstruction & progressive distention of the colon proximal to the affected segment. Acquired: chagas disease, in which trypanosomes directly invade the bowel wall to destoey the complexes. organic obstruction of the bowel neoplasm or inflammatory stricture. toxic megacolon complicating ulcerative colitis or crohn disease functional psychosomatic disorder ii. macroscopic Features Crohn disease Ulcerative colitis Bowel region Ileum & colon Colon only Distribution lesion Skip lesion Diffuse Stricture Early Late/rare

Edited by Foxit Reader Copyright(C) by Foxit Corporation,2005-2009 For Evaluation Only.

KHIZZER KHAN

Wall appearance Thickened Thin Dilation No Yes Microscopic: Pseudopolyps None to slight Marked Ulcer Deep linear Superficial Lymphoid reaction Marked Mild Fibrosis Marked Mild Serositis Marked Mild to none Granulomas Yes (40-60%) No Fistulas/sinuses Yes No Clinical: Fat/vitamin Yes No malabsorption Malignant potential Yes Yes Response to surgery Poor Good Q.NO. 21.A 30 years old female presented with sever pain in right iliac fossa. The is also history of fever & vomiting. (class test szmc.ryk) a. what is mostly diagnosis? b. enumerate four disease as differential diagnosis? c. write down the morphological findings. a. appendicitis b. 1. gastroenteritis with mesenteric adenitis 2. ruptured follicular cyst 3. rupture ectopic pregnancy 4. meckel diverticulum 5. mesenteric lymphadenitis with a viral systemic infection 6. PID with tubo-ovarian involvement c. morphology: neutrophilic infiltration of the muscularis propria serosal vessels are congested ulceration necrosis (acute suppurative appendicitis hemorrhagic green ulceration of the mucosa green black gangrenous necrosis (acute gangrenous appendicitis) Q.NO.22. Give TMN classification of colon cancers. Tumor (T) T0 none evident Tis in situ T1 invasion of lamina propria T2 invasion of muscularis propria T3 invasion in nonperitonealized perimuscular tissue T4 invasion of other organs or structure Lymph Node:

Edited by Foxit Reader Copyright(C) by Foxit Corporation,2005-2009 For Evaluation Only.

0 none evident 1 1-3 positive pericolic nodes 2 4 or more positive pericolic nodes 3 any positive nodes along a named blood vessels Distant metstasis: 0 none evident 1 any distant metastasis

KHIZZER KHAN

Edited by Foxit Reader Copyright(C) by Foxit Corporation,2005-2009 For Evaluation Only.

Chapter 18

KHIZZER KHAN

The Liver, Gallbladder & Biliary

Tract

Q.NO.1.A 40 years old obese female complaints of excruciating colicky (spasmodic) pain in right hypochondrium and dyspepsia. Her gall bladder reveals gall stones by ultrasonography. (annual 2011) a. what are the different types of gallstones? b. give the risk factors involved in the aetiology of gallstones. a. 1. cholesterol stone: (80%): containing crystalline cholesterol monohydrate 2. pigment stone: composed of predominantly bilirubin calcium salts b. risk factors for cholesterol stone: demography: northern Europeans, north and south Americans, native American advancing age female sex hormones: female gender, oral contraceptive, pregnancy obesity rapid weight reduction gallbladder stasis inborn disorders of bile acid metabolism hyperlipidemia syndromes Pigment stones: demography: Asian more then western, rural more than urban chronic hemolytic syndromes biliary infection GIT disorders: ileal disease (crohn disease), ileal resection or bypass cystic fibrosis with pancreatic insufficiency Q.NO.2.A biopsy from a 60 years old patient of chronic liver disease (hepa C positive) shows a nodular architecture. (annual 2010) a. what histological features are necessary for diagnosis of cirrhosis? b. what type of collagen is deposited in cirrhosis of liver and what cell type produce it? a. bridging fibrous septa in the form of delicate band linking portal tract with one another. Fibrosis is the key feature of progressive damage to liver. parenchymal nodules containing hepatocytes encircled by fibrosis. (Nodularity) disruption of architecture of entire liver b.collagen type 1 & 111 are deposited in the space of Disse, creating fibrotic septal defect

Edited by Foxit Reader Copyright(C) by Foxit Corporation,2005-2009 For Evaluation Only.

KHIZZER KHAN
Q.NO.3.A 20 years old medical student presents with symptoms of fever, nausea and vomiting and pain in right hypohondrium. There is history of passage of dark colored urine. History reveals that he had a meal at a road side restaurant three weeks ago. Laboratory investigations shows. (supp:2010) Serum: bilirubin: 8.5mg/dl (normal 0.2 to 1.2 mg/dl) Aspartate transaminase (AST): 800 U/L (normal upto 40 U/L) Alanine transaminase (ALT): 1200 U/L (normal upto 40 U/L) Alkaline phosphatase (ALP): 150 U//L (normal 50-120 U/L) Total protein 7.2 g/dl (normal 6.5 to 8.5 g/dl) Albumin 3,8 g/dl (normal 3.5-5.0 g/dl) Urine: bilirubin: positive & urobilinogen: positive a. what is the likely diagnosis? What does the raised transaminase indicate? b. Why does bilirubin & urobilinogen appear in urine in this condition? What form of bilirubin is present in a) plasma b) urine Q.NO.4.A 40 years old Taiwanese is admitted in medical ward with massive ascites. On examination two well defined solid nodules were detected in liver. Alpha fetoproteins level was 1000 ng/ml a. what could be the possible diagnosis? b. in your opinion which etiological factor is operative in this patient for producing this tumor? c. what is mechanism of ascites development in this patient? a. b. c. sinusoidal hypertension percolation of hepatic lymph in to the peritoneal cavity splanchnic vasodilation & hyperdynamic circulation Q.NO.5.A 50 year old lady presents with H/O dull right upper quadrant pain and flatulence for the past one year. Plain x ray revealed no stone in the gall bladder. Ultrasound was advised. Following the cholectectomy was performed. The lumen of gall bladder was found to full of stones. (annual 2009) a. enlist various types of gallstones? b. what are the expected microscopic features of removed gall bladder? c. what complications can develop in this gall bladder(if surgery was not performed)? a.see above

Edited by Foxit Reader Copyright(C) by Foxit Corporation,2005-2009 For Evaluation Only. KHIZZER KHAN

b. microscopy: c. 1. acute cholecystitis 2. pancrititis 3. blockage of bile duct 4. gall bladder cancer 5. empyema 6. cholangitis (inflammation of biliary tree) 7. perforation 8. fistula formation 9. intestinal obstruction (gall stone ileus or Bourverets syndrome) Q.NO.6. List two laboratory each for the evaluation of hepatocyte integrity, biliary excretory function and hepatocyte function. (annual 2008) 1. hepatocyte integrity: cytosolic hepatocellular enzymes serum aspartate aminotransferase (AST) serum alanine aminotransferase (ALT) serum lactate dehydrogenase (LDH) 2. Biliary excretory function: substance normally secreted in bile serum bilirubin: total, direct, delta(covalently linked to albumin urine bilirubin serum bile acids Plasma membrane enzymes serum alkaline phosphatase serum -glutamyl transpeptidase serum 5-nucleotidase 3. hepatocyte function: protein secreted in the blood serum albumin prothrombin time (factor v, vii, x prothrombin, fibrinogen) Hepatocyte metabolism: serum ammonia aminopyrine breath test galactose elimination Q.NO.7.a. What are three main types of liver disease associated with chronic consumption of alcohol? b. Give the salient histological features of each. (annual 2008) a. 1. hepatic steatosis 2. alcoholic hepatitis 3. cirrhosis b. 1. hepatic steatosis(fatty liver): microvascular lipid droplets in the hepatocytes

Edited by Foxit Reader Copyright(C) by Foxit Corporation,2005-2009 For Evaluation Only.

KHIZZER KHAN

with chronic alcohol intake clear macrovascular globules accumulate in cells perivenular fibrosis 2. alcoholic hepatitis: hepatocytes swelling & necrosis Mallory bodies Neutrophilic reaction fibrosis 3. cirrhosis: bridging fibrous septa in the form of delicate band linking portal tract with one another. Fibrosis is the key feature of progressive damage to liver. parenchymal nodules containing hepatocytes encircled by fibrosis. (Nodularity) disruption of architecture of entire liver Q.NO.8. A 60 years old male patient having chronic viral hepatitis B is discovered to have an 8cm mass lesion in right lobe of the liver. Serum AFP is markedly elevated. Give five important factors, which contribute to development of this tumor in patients of chronic hepatitis B. (supp: 2008) Q.NO.9. A 1-month old infant presenting with jaundice is diagnosed with extrahepatic biliary atresia (obstruction). (annual 2007) a. what are the major histological features on the liver biopsy? b. what two enzymes are likely to be raised in this condition? a. marked bile ductular proliferation portal tract edema fibrosis parenchymal cholestasis paucity of bile ducts b. 1. aminotransferases 2. alkaline phosphatase Q.NO.10. Write down the sequence of development of serologic markers of hepatitis B, viral infection, both acute and chronic. Strengthen your by making labeled graphic presentations. (annual 2006)

Edited by Foxit Reader Copyright(C) by Foxit Corporation,2005-2009 KHIZZER KHAN For Evaluation Only.

Q.NO.11.a. Enumerate the causes of jaundice. (annual 2004) b. how will you use laboratory tests to make diagnosis in a patient with jaundice? a. predominantly unconjugated hyperbilirubinemia excess production of bilirubin hemolytic anemia resorption of blood from internal hemorrhage ineffective erythropoiesis reduced hepatic uptake drug interference with membrane carrier system some cases of Gilbert syndrome impaired bilirubin conjugation physiologic jaundice of new born breast milk jaundice Predominantly conjugated hyperbilirubinemia Decreased hepatocellular excretion deficiency of canalicular membrane transporter ( rotor syndrome) Impaired bile flow primary sclerosing cholangitis b. 1. total bilirubin in serum 2. direct bilirubin 3. urine bilirubin 4. urine urobilinogen Q.NO.12. a Define cholelithiasis. b. classify gall stones. c. discuss aetiology and complications of gall stones. (supp: 2004) Q.NO.13. Give laboratory diagnosis of obstructive jaundice. (annual 2003)

Edited by Foxit Reader Copyright(C) by Foxit Corporation,2005-2009 For Evaluation Only. KHIZZER KHAN

Q.NO.14. A 50 year old lady presents with H/O dull right upper quadrant pain and flatulence for the past one year. Plain x ray revealed no stone in the gall bladder. Ultrasound was advised. Following the cholectectomy was performed. The lumen of gall bladder was found to full of stones. (SZMC. Sendup 2011) a. enlist various types of gallstones? b. what are the expected microscopic features of removed gall bladder? c. what complications can develop in this gall bladder(if surgery was not performed)?

KHIZZER KHAN

0300-710-3321

Edited by Foxit Reader Copyright(C) by Foxit Corporation,2005-2009 For Evaluation Only.

KHIZZER KHAN

Chapter 21

The Male Genital System

Q.NO.1. a. classify testicular germ cell tumors. (Annual 2011) b. give morphology of seminoma. a. classification: germ cell tumors seminomatous tumors: 1. seminoma 2. spermatocytic seminoma non-seminomatous tumors: 1. embryonal carcinoma 2. yolk sac (endodermal sinus) tumor 3. choriocarcinoma 4. teratoma sex cord stromal tumors: 1. leyding cell tumor 2. sertoli cell tumor b. morphology of seminoma: gross morphology: present as uniform solid enlargement of testis, the surface is smooth. on cut section: tumor has homogenous, gray white, lobulated appearance called potato tumor. tumor grow rapidly & entire testis is replaced generally, tunica albuginea is not penetrated but occasional extension to Epididymus, spermatic cord or sacrotal sac may occur Microscopic: composed of sheets or cord of uniform seminoma cells which are large, round to polyhyderal cells with distinct cell membranes, clear cytoplasm & central round nuclei the sheets or cords of cells are divided into poorly demarcated lobules by delicate fibrous septa which show lymphocytic infiltration & occasional granulomas. a few may contain syncytial giant cells that secrete HCG. Q.NO.2. A patient presents with palpable mass in abdomen and is found to have bilateral cryptorchidism. (annual 2010) a. list the testicular tumors that could be found in this patient.

Edited by Foxit Reader Copyright(C) by Foxit Corporation,2005-2009 For Evaluation Only.

KHIZZER KHAN b. Give THREE histological features of teratoma. a. germ cell tumors b. composed of well differentiated elements such as neural tissue, muscle, cartilage, squamous epithelium, etc in fibrous or myxiod stroma these elements are arranged in no definite pattern the foci of squamous cell carcinoma, mucin secreting adenocarcinoma may be seen. malignant teratoma may contain yolk sac or trophoblastic tissue that secrete AFP. Q.NO.3.A 70 years old man has increasing difficulty with urination. He also has a feeling of urgency, but each time the urine volume is small. He has difficulty in starting and stopping urination. His problem has progressed last few years. His prostatic specific antigen level is slightly increased but has been stable over this time. (supp:2010) a. what is the likely diagnosis? b. give the microscopic appearance of prostate in this condition. c. enumerate the secondary changes that can occur in lower urinary tract and kidney in this condition. a. Benign prostatic hyperplasia b. microscopic: results from glandular proliferation and composed of glands of variable size, lined by hypertrophic tall columnar epithelium which is thrown into numerous papillary bud & infoldings & contain numerous hyline concretion called corpora amylasea the glands are separated by stroma infiltrated with lymphocytes. some time the hyperplasia is predominantly fibromuscular & contain solid masses spindle cells. foci of ischemic necrosis surrounded by squamous metaplasia involving the glands may be seen. c. Q.NO.4. a. Name any four germ cell tumors of the testis. Enumerate the biological markers secreted by such tumors that can be detected in blood. b. Enumerate three predisposing influences that contribute to the pathogenesis of germ cell tumors of testis? (annual 2009) a. biological marker: 1. alpha fetoprotein (AFP) 2. Human chorionic gonadotropin (HCG)

Edited by Foxit Reader Copyright(C) by Foxit Corporation,2005-2009 KHIZZER KHAN For Evaluation Only.

b. 1. genetic factors: more in blacks & jews. 2. endocrinal factors 3 environmental factors: carcinogens 4. abnormalities: cryptorchidism, gonadal dysgenesis Q.NO.5.A 54 years old male develops frequency of micturation, urgency, overflow incontinence and nocturia. Digital rectal examination reveals nodular prostatic enlargement. Serum PSA is <4ng/dL. Sextant prostate biopsies are negative for malignancy: (annual 2008) a) what is the diagnosis? b) if a prostatectomy were done in this patient, what gross and microscopic features would you expect to find? a. nodular hyperplasia of the prostate b. morphology: gross prostate shows nodules which weigh 300gm in sever cases. in lateral lobe enlargement, urethra is compressed to slit like orifice. while in middle lobe enlargement, a hemispherical mass projects under the floor of bladder producing at times a ball valve effect. on cross section, nodules; - arise around the urethra from inner prostatic mass surrounded by a false capsule - yellow pink & soft when hyperplasia is mainly glandular, while pale gray & fibrous when hyperplasia is mainly fibromuscular. Microscopic: results from glandular proliferation and composed of glands of variable size, lined by hypertrophic tall columnar epithelium which is thrown into numerous papillary bud & infoldings & contain numerous hyline concretion called corpora amylasea the glands are separated by stroma infiltrated with lymphocytes. some time the hyperplasia is predominantly fibromuscular & contain solid masses spindle cells. foci of ischemic necrosis surrounded by squamous metaplasia involving the glands may be seen. Q.NO.6.a.What three serum markers would you order in a patient with a testicular mass? b. what is the value of serum markers in the context of testicular tumors? (Annual 2007) a. 1. hCG.2. AFP 3. Lactate dehydrogenase seminoma: 10% have elevated hCG embryonal carcinoma: 90% elevated both hCG & AFP

Edited by Foxit Reader Copyright(C) by Foxit Corporation,2005-2009 For Evaluation Only.

KHIZZER KHAN Yolk sac tumor: 90% have elevated AFP choriocarcinoma: 100% have elevated hCG teratoma: 50% have both elevated mixed tumor: 90% have both elevated Q.NO.7. Classify testicular tumors. Give briefly morphology of teratoma. (Annual 2006) a. *tumor with one histological pattern: seminoma embryonal carcinoma yolk sac carcinoma choriocarcinoma teratomas: ( mature, immature, with malignant transformation of somatic elements) *Tumor with more than one histologic pattern: Together grouped as non-seminomatous tumors Gross morphology: presents as irregular & nodular enlargement of testits on cut section: - shows cystic areas - appearance is is variegated with foci of cartilage, bone or soft myxomatous tissue. dermoid cysts are rare in the testis Microscopic: composed of well differentiated elements such as neural tissue, muscle, cartilage, squamous epithelium, etc in fibrous or myxiod stroma these elements are arranged in no definite pattern the foci of squamous cell carcinoma, mucin secreting adenocarcinoma may be seen. malignant teratoma may contain yolk sac or trophoblastic tissue that secrete AFP. Q.NO.8. Describe the morphology of seminoma. Enumerate the various types of germ cell tumor of testis. (Annual 2005) Morphology of seminoma: See Q.NO.1 Q.NO.9. Give gross and microscopic features of mature teratoma. (Annual 2004) See Q.NO.7

Q.NO.10. write short notes on: seminoma testis. (2003) See Q.NO.1 Q.NO.11. a. classify testicular germ cell tumors. (SZMC. Sendup 2011) b. give morphology of seminoma.

Edited by Foxit Reader KHIZZER KHAN Copyright(C) by Foxit Corporation,2005-2009 For Evaluation Only.

Chapter 22

The Female Genital System & Breast


Q.NO.1. a. Classify ovarian tumours. (annual 2011) b. give pathogenesis of ovarian tumors. a. surface epithelial stromal cell tumors: 65-70%, age : 20+ years serous tumor mucinous tumor endometroid tumor clear cell tumor transitional cell carcinoma epithelial stromal (adenosarcoma, malignant mixed mullerian tumor) Germ cell derived tumors: 15-20%, age:0-25+ years teratoma (immature, mature, solid, cystic adenoid, monodermal) dysgerminoma endodermal sinus tumor (yolk sac tumor) choriocarcinoma sex cord stromal tumors: 5-10% : all ages fibroma granulosa theca cell tumor sertoli- leyding cell tumor sex cord tumor with annular tubules gynandroblastoma steroid ( lipid ) cell tumors metastatic cancer colonic, appendiceal gastric breast b. majority of ovarian cancers seemed to be caused by mutations in the BRCA genes, BRCA1 or BRCA2 Risk factors: 1. nulliparity 2. family history 3. prolonged used of OCP reduced risk 4. early menarche 5. pelvic irradiation Q.NO.2.An 18 years old female presents with solitary, discrete, movable mass in right breast which enlarges late in the menstrual cycle. FNAC is performed which shows a benign breast lesion. (annual 2011) a. what is the likely diagnosis? b. give morphological details of the lesion. a. fibroadenoma b. gross: presents as spherical, solitary nodules that is sharply circumscribed & moveable size varies from 1-10 cm in diameter called giant fibroadenoma occurs mostly in upper quadrant of breast

Edited by Foxit Reader KHIZZER KHAN Copyright(C) by Foxit Corporation,2005-2009 For Evaluation Only.

on cut section, it is firm & grayish white Microscopic: composed of loose fibroblastic stroma containing glandular & cystic spaces lined by single or multiple layers of the cells glandular spaces may be open, round to oval & regular ( pericanalicularfibroadenoma) or compressed, irregular clefts due to extensive stromal proliferation, ( intracanalicular fibroadenoma) Q.NO.3 A cervical biopsy from a 35 years old sexually active female is submitted to you for reporting; previous PAP smear exam has shown low grade intraepithelial neoplasia on two occasions and high grade intraepithelial lesion subsequently. (Annual 2010) a. outline the grading system you would use histologically classify the cervical epithelial lesion in this patient. b. what are TWO commonest HPV genotype associated with high grade cervical intraepithelial lesion? a. CIN 1: mild dysplsia. Lower one third epithelium is involved pleomorphism hyperchromatic mitosis above the basal layers architectural anarchy of cells kiolocytes are seen CIN 11: moderate dysplasia CIN 111: sever dysplasia & carcinoma in situ b. HPV 16 & 18 Q.NO.4.A top tennis player was diagnosed with fibrocystic disease of the breast with atypical ductal hyperplasia. Routine mammography showed an area of increased density a. how does this diagnosis affect the patients risk of developing breast cancer? b. what other factors will modify (increase or decrease) this risk? c. define atypical hyperplasia? (annual 2010) a. the relative risk of developing of carcinoma in fibrocystic disease is 1 (3%) but due to atypia risk increased by 4 to 5 (13% to 17%) b. increased risk: 1. early menarche 2. radiation exposure 3. nulliparous woman 4. tobacco 5. environmental toxin risk decreased by: 1. bilateral prophylactic mastectomy 2. estrogen antagonist (tamoxifen) c. atypical hyperplasia: the hyperplastic cell become monochromic with complex architectural pattern. They have changes approaching those of ductal carcinoma in situ. Such hyperplasia is called atypical

Q.NO.5. A 45 years old woman feels pressure sensation, but no pain in pelvic region for the past 6 months. On examination there is right adnexal mass. An ultrasounds scan shows a 10 cm fluid filled cystic in the right ovary. A fine needle aspirate is performed

Edited by Foxit Reader KHIZZER KHAN Copyright(C) by Foxit Corporation,2005-2009 For Evaluation Only.

and cytological examination of clear fluid aspirate from the mass reveals clusters of malignant cells surrounding psammoma bodies. (Supp:2010) a. what is the likely diagnosis? b. what are the gross and microscopic appearance of this tumor? a. ovarian papillary serous cystadenocarcinoma b. gross: shape is spherical to ovoid cyst surface shows nodular irregularities on cut section: - small cysts are unilocular, while large cysts are multilocular - filled with clear serous fluid - papillary projections are present in the cavities Microscopic: psammoma bodies are present in the tips of papillae anaplasia of the lining cells papillary formation are complex & multilayered invasion of stroma Q.NO.6.A 60 year old female presents with a hard lump in the breast. It was excised and diagnosed a invasive ductal carcinoma breast. No special type. (Supp:2010) a. describe morphology. b. what further investigations will you suggest on the biopsy tissue before start the treatment in this patient? a. gross: presents as delimited mass, rarely over 3-4cm in dia, of stony consistency (hence the name scirrhous CA) Extension of the tumor may cause dimpling of skin, retraction of nipple & fixation to chest wall on cut section: - tumor is infiltrative & retracted below the surrounding fibrofatty tissue . (crabe-like shape) - color is grayish, white with yellow specks ( unripe pear appearance) - foci of chalky white necrosis & calcification may be seen. Microscopic: composed of dense fibrous stroma in which anaplastic duct lining cells disposed in nests, cords, tubes or gland like pattern neoplastic cells are round to polygonal with small dark nuclei, few mitotic figures neoplastic cells are seen infiltrating surrounding tissue, pervascular & perineural space as well as blood vessels. b. Following molecular classes correlate with prognosis & response to therapy. So we classified before starting the treatment: 1. luminal A: (40-50% of NST CA). ER + & HER2/neu negative ..chemotherapy 2. luminal B: (15-20%) ER & HER2/neu +, high proliferative rate: triple + cancer.. response to chemotherapy. 3. normal basal like: (6-10%)ER + & HER2/neu negative 4. basal like: (13-25%): ER,PR & HER2/neu are negative : triple negative 5. HER2/neu positive: (7-12%): ER negative. Rx (Trastuzumb)

Edited by Foxit Reader Copyright(C) by Foxit Corporation,2005-2009 KHIZZER KHAN NMC For Evaluation Only.

Q.NO.7.A 28 years old woman goes to her physician for routine examination. Pelvic examination reveals no abnormalities. A pap smear report indicates the presence of severely dysplastic cells. A cervical biopsy shows cervical intraepithelial neoplasia (CIN-111). (Annual 2009) a. Give the important risk factors for the development of CIN & invasive carcinoma of cervix. b. Give the spectrum of precancerous lesions of the cervix based on histology, progressing to carcinoma. c. based on the cell of origin, enumerate the most common cervical carcinomas. a. b. c. 1. epithelial tumors: benign: squamous cell papilloma malignant: squamous cell carcinoma adenocarcinoma adeno-squamous CA 2. non epithelial tumor: leiomyoma Lieomyosarcoma 3. miscellaneous tumors: mullerian mixed tumors 4. secondary tumors: squamous metaplasia Glandular hyperplasia Endometriosis Q.NO.8.A 45 years old lady complaints of a lump in right breast which is hard in consistency. (annual 2009) a) which relatively non-invasive investigation will you advice in order to evaluate benign or malignant nature of lesion? b) what are the prognostic markers of carcinoma breast? a. 1. mammography 2. ultrasonography 3. MRI b. major prognostic markers: 1. invasive carcinoma versus in situ disease 2. distant metastasis 3. lymph nodes metastasis 4. tumor size 5. locally advanced disease 6. inflammatory carcinoma Minor prognostic & predictive factors: 1. histologic subtype 2. histologic grade 3. estrogen/progesterone receptors 4. HER2/neu 5. proliferative rate

Edited by Foxit Reader Copyright(C) by Foxit Corporation,2005-2009 For Evaluation Only.

KHIZZER KHAN NMC

6. DNA content 7. response to non-adjunctive therapy 8. gene expression profile Q.NO.9. A 45 years old female with multiple sexual partners has cervical PAP smear done as part of routine screening for cervical cancer. The smear is reported as CIN 11 with advice for further workup: (annual 2008) a) screening for DNA of which virus is most pertinent to the current scenario? b) what role does the said virus play in pathogenesis of cervical cancer? a. Human papilloma virus (HPV) b. HPV types 16 & 18 usually integrate into the host genome & express large amounts of E6 & E7 proteins which block or inactivate tumor suppressor genes p53 & RB respectively. The result in transformed cell phenotype, capable of autonomous growth & susceptible to the acquisition of further mutation. Q.NO.10.A mastectomy specimen from a 42 years old female shows a 2 cm tumor which is a well differentiated ductal carcinoma of no special type with a minor in situ component. Two of 14 axillary lymph nodes are positive for metastatic tumor. The tumor is estrogen receptor and progesterone receptor negative and her 2-neu positive. There is no distant metastasis. (annual 2008) a. which of these tumor characteristic has the greatest effects on clinical outcome in this patient? b. identify all prognostic and predictive factors listed here and give relative importance of each? a. pathologic examination of the primary carcinoma & axillary lymph nodes b. 1. tumor size: if tumor size is 1cm then excellent prognosis but in this case 2cm so poor prognosis 2. lymph nodes involvement: most important prognostic factor - if no node is involved: survival rate 70-80% - one to three positive: survival rate 35-40% - if more than 10 positive: 10-15% survival rate 3. distant metastasis: no distant metastasis so more curable 4. HER2/NEU: is predictor of response to agents that target this transmembrane Protein (e.g. trastazumab, lapatinib). Over expression poor survival. 5. estrogen/progesterone receptors: it is negative so poor response to the therapy So poor prognosis. Q.NO.11.A 55 years old female has a benign cystic ovarian epithelial neoplasm: a. what are two major types of these tumors? b. how do they differ in their epithelial lining? c. list three major categories of primary ovarian neoplasms according to their cell/tissue of origin. Give one example of each. (supp:2008) a. 1. serous tumor 2. mucinous tumor b. c. surface epithelial stromal cell tumors: 65-70%, age : 20+ years

Edited by Foxit Reader Copyright(C) by Foxit Corporation,2005-2009 For Evaluation Only.

KHIZZER KHAN NMC

serous tumor mucinous tumor endometroid tumor clear cell tumor transitional cell carcinoma epithelial stromal (adenosarcoma, malignant mixed mullerian tumor) Germ cell derived tumors: 15-20%, age:0-25+ years teratoma (immature, mature, solid, cystic adenoid, monodermal) dysgerminoma endodermal sinus tumor (yolk sac tumor) choriocarcinoma sex cord stromal tumors: 5-10% : all ages fibroma granulosa theca cell tumor sertoli- leyding cell tumor sex cord tumor with annular tubules gynandroblastoma steroid ( lipid ) cell tumors metastatic cancer colonic, appendiceal gastric breast Q.NO.12. a. list four major and two minor risk factors for breast cancer. (supp:2008) b. list the evolving morphologic spectrum of lesion that eventually result in invasive carcinoma of breast. a. major: geographic factor: more common in North America & Europe than Asia. age: increases after age 30yr genetic & family history: first degree relative with breast cancer menstrual history: age at menarche< 12yr & delayed menopause pregnancy: more chances in nulliparous benign breast disease: proliferative lesion Minor factors: prolonged exposure to exogenous estrogens oral contraceptive obesity high fat diet \ alcohol consumption cigarette smoking ionizing radiation b. morphology: 1. these include a tendency to become adherent to the pectoral muscles or Deep fascia of the chest wall with consequent fixation of the lesion 2. adherence to the overlying skin with retraction or dimpling of the skin of the nipple. Important sign observed by woman herself during self exam. 3. involvement of the lymphatic pathway may cause local lymphedema.

Edited by Foxit Reader Copyright(C) by Foxit Corporation,2005-2009 For Evaluation Only.

KHIZZER KHAN NMC

4. the skin become thickened around exaggerated hair follicles, a change known as peau dorange (orange peel) Q.NO.13.A 45 years old female patient develops a peanut sized nodules in an old midline laparotomy scar, which becomes painful during menstrual period. The excised nodule consists of normal looking endometrial tissue with glands and stroma. (Annual 2007) a. give three theories of pathogenesis of such lesion s. b. list four important sites for this process other than the abdominal wall?. a. endometriosis 1. regurgitation theory, follapian tube 2. metaplastic differentiation of coelomic cavity 3. vascular or lymphatic dissemination theory b. 1. lungs 2. heart 3. skeletal muscles 4. bones Q.NO.14. a. pathologist is grading a breast tumor according to the Scarf-BloomRichardson system. What three morphological features will he assess? (Annual 2007) b. what is the significance of ER/PR and Her 2-neu status in a breast carcinoma? a.1. tubules formation 2. nuclear grade 3. mitotic rate: i. well differentiated ii. Moderately differentiated iii. Poorly diffb. overexpression of this membrane bound protein is almost always caused by amplification of the gene. Overexpression can be determined by immunohistochemistry (which detects the protein in tissue section) or by fluorescence in situ hybridization (which detects the number of gene copies). Over expression is associated with poor prognosis. However the importance of evaluating HER2-NEU is to predict response to a monoclonal antibody (Herceptin) to the gene product. This is one of the first example whereby an antitumor antibody therapy has been developed on the basis of specific gene abnormality present in the tumor. Q.NO.15. a. Classify breast tumors. (Annual 2006) b. Give the morphology of lobular carcinoma. a. benign tumors: fibroadenoma phyllodes tumor intraductal pipilloma intraductal papilloma Carcinomas: Non-invasive: DCIS (ductal carcinoma in situ). Paget disease of nipple is extension of DCIS. LCIS (lobular carcinoma in situ) Invasive: invasive ductal carcinoma invasive lobular carcinoma medullary carcinoma colloid carcinoma tubular carcinoma

Edited by Foxit Reader Copyright(C) by Foxit Corporation,2005-2009 For Evaluation Only.

KHIZZER KHAN NMC


other types: e.g ductal ( scirrhous carcinoma) b.Morphology of lobular carcinoma: tumor comprises fewer than 20% of all breast cancer the cells invade individually into stroma and often aligned in strands or chain surround cancerous or normal appearing acini&ducts creating so called bull eye pattern most presents as palpable masses or mammographic densities these carcinomas more frequently than ductal carcinoma metastasize CSF, GIT etc. lobar carcinomas are more frequently multicentric & bilateral (10-20%) all these carcinomas express hormone receptors but HER2/NEU overexpression is rare. Q.NO.16. Name the various risk factors of carcinoma cervix. Give clinical staging of carcinoma cervix. Risk factors: early age at first intercourse multiple sexual partner a male partner with multiple previous sexual partners lower socioeconomic group rarity among virgin smoking immunocompromised woman persistent infection with a high oncogenic risk HPV e.g HPV 16 or HPV 18. High parity oral contraceptive certain HLA subtypes Clinical staging: Stage 0 Carcinoma in situ Stage 1 Carcinoma confined to cervix 1a Preclinical carcinoma, diagnosed only by microscopy 1a1 Stromal invasion no deeper than 3mm and no wider than 7mm (microinvasive) 1a2 Maximum depth of invasion of stroma deeper than 3 mm and 7mm horizontal 1b Invasion Greater than stage 1a2 Stage11 Carcinoma extends beyond the cervix but not to the pelvic wall, vagina etc Stag111 Carcinoma has extend the pelvic wall, involves the lower third of vagina Stage1V Carcinoma has extended beyond the true pelvis, involves bladder or rectum. Q.NO.17. a. Enumerate epithelial tumors of the ovary? (Annual 2005) b. Describe the types of hydatidiform mole. a. b. complete (classic) Mole: all the villi are edematous & there is diffuse trophoblastic proliferation epithelial cells are diploid (46XX/46XY) It results from either fertilization of empty egg by a single sperm & the duplicates or from the fertilization of empty by two sperms embryo dies early & thus complete mole has no fetal parts Partial Mole: some villi are edematous & there is focal trophoblastic proliferation epithelial cells are triploid (69XXX, XYY,XXY),even occasionally tetraploid (99xxxy) it results from the fertilization of an egg with two sperm

Edited by Foxit Reader KHIZZER KHAN NMC Copyright(C) by Foxit Corporation,2005-2009 For Evaluation Only.

embryo is viable for weeks & thus partial mole has fetal parts Morphology: usually discovered in 4th-5th month of pregnancy & uterus is large for dates uterine cavity is filled with a mass of delicate, thin walled translucent, grape like shape Microscopic: Complete mole: hydropic swelling of all the chrionic villi diffuse trophoblatic proliferation absence of vessels, edematous stroma Partial mole: hydropic swelling of some of the villi focal trophoblastic proliferation Features Complete mole Partial mole Karyotype 46, XX, XY Triploid (69, XXY) Villous edema All villi Some villi Trophoblast proliferation Diffuse Focal Atypia Often present Absent Serum Hcg Elevated Less elevated hCG in tissue +++ + Behavior 2% choriocarcinoma Rare choriocarcinoma Q.NO.18.write short notes on: fat necrosis breast (annual 2005) Painless palpable mass, skin thickening or retraction, a mammographic density. The majority of women have history of breast trauma or prior surgery. Major clinical significance of this condition is possible confusion with breast cancer. Liquefactive fat necrosis is present. Q.NO.19. Enlist ovarian neoplasms. (annual 2004) a. * germ cell lesion : Tumors Peak incidence Usual location Morphological features Behavior Dysgerminoma Second to third 80 to 90 % Counterpart of testicular All malignant but decades, occur Unilateral seminoma. Solid large to only one third with gonadal small gray masses. Sheets aggressive and dysgenesis or cords of large cleared spread: all cell separated by scant radiosensitive with fibrous strands. Stroma 8% cure may contain lymphocyte and occasional granuloma Choriocarcinoma First three Unilateral Identical to placental Metastasis early decades of life tumor. Often small, and widely. hemorrhagic focus with Primary focus my two type of epithelium; disintegrate cytotrophoblast & leaving only mets syncytiotrophoblast Sex cord tumors: Peak Usual

Tumors

Morphology

Behavior

Edited by Foxit Reader KHIZZER KHAN NMC Copyright(C) by Foxit Corporation,2005-2009 For Evaluation Only.

Granulose thecal cell

incidenc Most postmen opausal age Any age

location Unilateral

Thecoma fibroma Sertolileyding cell

Unilateral

May be tiny or large. Gray to yellow. Composed of mixture of granulose cells in cords, sheets or strands and spindled or plump lipid laden thecal cells. Granulosal element may recapitulate ovarian follicle as Call Exner bodies Solid gray fibrous cells to yellow plump thecal cells

Usually small, gray to yellow brown and solid. Recaps development of testis with tubules or cord and plump pink sertoli cell Other ovarian tumor: teratomas

All age

Unilateral

May elaborate large amounts of estrogen and so may promote endometrial or breast carcinoma Most hormonally inactive. Rarely malignant Many masculinizing or defeminizing. Rarely malignant

Q.NO.20. a. Name the benign and malignant neoplasms of the breast. (supp:2004) b. give gross and microscopic features of fibroadenoma of breast. a. b. gross: fibroadenoma occurs as discrete, usually solitary free moveable nodules 1-10cm in diameter. If they exceed 10 cm in diameter (giant fibroadenoma) all are firm, with a uniform tan white color in cut section punctuated by softer yellow pink specks representing the glandular mass. Microscopy: composed of loose fibroblastic stroma containing glandular & cystic spaces lined by single or multiple layers of the cells glandular spaces may be open, round to oval & regular ( pericanalicularfibroadenoma) or compressed, irregular clefts due to extensive stromal proliferation, ( intracanalicular fibroadenoma) Q.NO.21. a) classify tumors of breast. (annual 2003) b) give the morphological features of fibro-adenoma breast. C) give the outline of pathogenesis of breast cancer associated with different Receptors pertaining to breast in females Q.NO.22. Enumerate hormones secreting tumours of ovary. (annual 2003) a. sex cord stromal tumors b. germ cell tumors Q.NO.23. a. what is the pagets disease of nipple? (supp: 2003) b. endometrial hyperplasia c. benign cystic teratoma ovary a. gross: the nipple & aerola is eczematiod, fissured, ulcerated & oozing there is surrounding inflammatory hyperplasia & edema an underlying lump is rarely present occurs in older age group. Superimposed bacterial infection is common Microscopic: characteristic feature is invasion of epidermis by neoplastic cell called paget cells.

Edited by Foxit Reader KHIZZER KHAN NMC Copyright(C) by Foxit Corporation,2005-2009 For Evaluation Only.

These are large, hyperchromatic cell surrounded by a clear halo which represents intracellular accumulation of mucopolysaccrides. basal epithelial cells are compressed & underlying dermis shows plasma cell infiltration. features of intraductal CA are also present because pigets disease begins as intraductal CA which later involves main excretory ducts & extends to infiltrate the skin of nipple. b. endometrial hyperplasia: Definition: pathological hyperplasia in which there is proliferation of the endometrium. Etiology & pathogenesis: is caused by relative or absolute hyper-estrinism such as seen in 1. polycystic ovaries 2. chronic failure or ovulation 3. estrogen secreting ovarian tumors 4. adenocortical hyperfunction 5. prolonged use of exogenous estrogen morphology: gross: uterus is enlarged endometrium is thickened mayometrium is also thickened Microscopic: there are three subtypes simple hyperplasia with out atypia: (mild or cystic hyperplasia) complex hyperplasia: (moderate or edematous hyperplasia) atypical hyperplasia: ( complex hyperplasia with atypia) c. benign cystic teratoma of the ovary: benign cystic teratomas are marked by ectodermal differentiation & line by normal appearing skin with its associated adnexal appendages occur b/w 20-30years of age Gross morphology: shape is spherical to ovoid cyst size is less than 10cm in diameter surface is covered by glistening serosa occurs usually unilateral, mostly on right side on cut section: - unilocular cyst lined by normal appearing skin with it adnexal appendages esp. hair - filled with thick sebaceous secretion containing matted hair - teeth protruding from nodular projection (dermoid cyst) Microscopic morphology: lined by well differentiated skin with its adnexal appendages esp. hair foci of bone, cartilage, bronchial or gut epithelium are present in the nodular projection Q.NO.24. 40 years old female presented in gynae OPD with complaints of menorrhagia & abdominal mass. On USG diagnosis of leiomyoma is made. (SZMC sendup 2011) a. what histopathologic findings are noted in biopsy specimen? b. enumerate other sites where this tumor may be found.

Edited by Foxit Reader Copyright(C) by Foxit Corporation,2005-2009 For Evaluation Only.

KHIZZER KHAN NMC

Chapter 24

The Endocrine System

Q.NO.1.A 30 years old female presents with soft, warm and flushed skin. Heat intolerance and excessive sweating. She gives history of weight loss despite increase appetite & frequent attacks of diarrhea, palpitation and experiences nervouness, tremor and irritability. On examination she has a wide, staring gaze and lid lag are present. She is provisionally diagnose to have thyrotoxicosis. (annual 2011) a. what is triad of manifestations in grave disease? b. what are the morphological features of a thyrotoxic thyroid? a. 1. thyrotoxicosis 2. ophthalmopathy 3. dermopathy b. gross: 1. diffused hypertrophy & symmetrical enlarge 2. On cut section: -Soft meaty appearance resembling normal muscles Microscopic: epithelium tall & more corroded, small papilli projecting into the follicular . lumen, pale colloid, lymphoid infiltrate with germinal centre. Q.NO.2.A 35 years old female presents with generalized apathy, mental sluggishness, restlessness, cold intolerance and obesity. She is suspected to have myxedema. a. what lab tests will you order to confirm the diagnosis? (annual 2011) b. what lab findings are going to be present if she is diagnosed to have hashimoto thyroiditis? c. why some patients with graves disease spontaneously developed episodes of hypothyroidism? a. TSH level . increased in primary hypothyroidism T4 Level T3 level b. antimicrosomal & antithyroglobulin antibodies c. TSH-binding inhibitor immunoglobulins (TBII): These anti-TSH receptor antibodies prevent TSH from binding normally to its receptors on thyroid epithelial cell. Some forms of TBBIIs mimic the action of TSH and resulting in the stimulation of thyroid epithelial cell activity. Whereas other forms may actually inhibit the thyroid cell function. It is not usual to find the coexistence of stimulating and inhibiting immunoglobulins in the serum of the same patient, this finding explain

Edited by Foxit Reader KHIZZER KHAN NMC Copyright(C) by Foxit Corporation,2005-2009 For Evaluation Only.

why some patient with grave disease spontaneously develop episodes of hypothyroidism. Q.NO.3.A 45 years old woman presents with painless enlargement of thyroid. On examination her physical and mental responses appear sluggish. Histological examination of thyroidectomy specimen shows lymphocytic infiltrate with germinal centre formation along with thyroid follicles, few showing Hurthle cell change. (an2010) a. give diagnosis and immunological mechanism of cell injury. b. name two endocrine diseases associated with this condition. a. hishimoto thyroiditis pathogenesis: the possible reaction of CD4+ T cells to thyroid antigens, thus producing cytokines Interferon (IFN-). Which promote inflammation & activate macrophages, as in Delayed type hypersensitivity reactions. CD8+ cytotoxic T cell mediated cell death: binding of antithyroid antibodies followed by antibody dependent cell mediated cytotoxicity mediated by natural killer cells

b. 1. subacute granulomatous thyroiditis 2. subacute lymphocytic thyioiditis Q.NO.4. Give laboratory findings of serum T3, T4 & TSH level in typical cases of Graves disease, Hashimotos thyroiditis; diffuse non-toxic goiter and multinodular goiter. (annual 2010) Graves disease: TSH: T3:

Edited by Foxit Reader Copyright(C) by Foxit Corporation,2005-2009 For Evaluation Only.

Hishimoto thyroiditis:

T4: initial phase TSH T3 T4 Delayed phase TSH: T3: T4:

KHIZZER KHAN NMC

Diffuse & multinodular : TSH or normal compensatory increased Goiter T3 or normal T4 or normal Q.NO.5. A 40 years female presents with enlarge nodular thyroid. Her thyroid function tests were: (supp:2010) TSH: 8.0 mUI/L (normal: 0.20-4.0 mUI/L) T3 : normal T4 : 30 nmol/l (normal 55-160 nmol/l) a.what other relevant investigation can you demand in this patient and what is the expected result? b. what will be the microscopic morphology of thyroidectomy specimen? How does they correlate with the underlying thyroid pathology? a. radioactive iodine uptake

Q.NO.6.An 18 years old known diabetic girl, suddenly felt ill, developed vomiting and became drowsy. She was taken to the emergency, where on examination she had B.P of 95/60mm of Hg with pulse rate 112/min and cold extremities. She had deep, sighing respiration (kussuauls respiration) and her breath smelt of acetone. (annual 2009) a. what is the likely diagnosis? b. give any four clinical and four metabolic features of this condition. a. diabetic ketoacidotic coma b. sever insulin deficiency inc lipolysis increase free fatty acid level inc FFA oxidation in liver ketone bodies formation ketonemia and ketonuria acidosis diabetic coma 1. acetone breath 2. osmotic diuresis 3. dehydration 4. ketonemia, ketoneuria

Edited by Foxit Reader Copyright(C) by Foxit Corporation,2005-2009 For Evaluation Only. KHIZZER KHAN NMC

5. metabolic acidosis Q.NO.7. Write a short note on insulin resistance. (annual 2009) The resistance to the effects of insulin on glucose uptake, metabolism and storage is called insulin resistance. It is characteristic feature of type 2 diabetes. The evidence that insulin resistance has a major role in the pathogenesis of type 2 D.M can be gauged from the findings that: 1. Insulin is often detected 10 to 20 years before the onset of diabetes in predisposed Individuals (offspring of type 2 diabetic). 2. in prospective studies the insulin resistance is the best predictor for subsequent progression to diabetes. genetic defect of insulin receptor and insulin signaling pathway obesity and insulin resistance

Q.NO.8.A biopsy from soft tissue mass in the iliac fossa revealed metastatic carcinoma most consistent with a diagnosis of metastatic follicular carcinoma of thyroid gland. A search into previous record of

Edited by Foxit Reader Copyright(C) by Foxit Corporation,2005-2009 KHIZZER KHAN NMC For Evaluation Only.

the patient showed history of nodulectomy for a 2cm cold thyroid nodule 8 years back. The lesion was reported as follicular adenoma(Annual2008) a. if you were to re-examine the slides from the thyroid nodules which part of the lesion . would you examine most carefully and why? b. what is most common mode of metastasis for follicular thyroid cancer and what are the commonest site for metastasis? a. Capsule ..to distinguish it from the follicular adenoma ..to exclude capsular or vascular lesion. b. hematogenous spread : lungs, liver, bones Q.NO.9. A 32 years old female patient presents with visual disturbance, hirsutism and moon facies. 24 hours urinary free cortisol levels are elevated. CT scan shows a space occupying lesion in the pituitary gland impinging on the optic chiasm: (supp:2008) a. what is the diagnosis? b. list two other aetiological types of this disease. c. how will you determined the etiological cause of this disease in the lab if no radiological data is available? a. cushing disease b. cushing syndrome, Ectopic ACTH production from tumor of any organ. E.g small cell carcinoma of lungs. c. Administration of dexamethasone normally will suppress pituitary ACTH production , resulting in suppression of adrenal cortisol production & decrease in urinary free cortisol. But in case of ectopic & adrenal cushing ACTH will not suppress. Q.NO.10. Write down the pathogenesis of the graves disease. (supp:2008) Graves disease is autoimmune disorder in which a verity of antibodies may be present in the serum, Including antibodies to the TSH receptor, thyroid peroxisomes & thyroglobulin: 1. Thyroid stimulating immunoglobulin (TSI): 2. thyroid growth stimulating immunoglobulins (TGIs): 3. TSH binding inhibitor immunoglobulins (TBBII): Q.NO.11. a. While examining H & E sections from a thyriodectomy specimen, a pathologist notices a lesion having a follicular pattern of growth. What is the differential diagnosis of such lesions? (annual 2007) b. list the three nuclear features diagnostic for pipallary carcinoma of thyroid. a. 1. follicular adenoma 2. follicular carcinoma 3. papillary carcinoma (follicular variant)

Edited by Foxit Reader Copyright(C) by Foxit Corporation,2005-2009 For Evaluation Only. KHIZZER KHAN NMC

b. 1. nuclei of papillary carcinoma cells contain very finely dispersed chromatin 2. this chromatin imparts an optically clear appearance, giving rise to designation ground glass or orphan annie eye nuclei 3. invaginations of cytoplasm (pseudo-inclusions) 4. psammoma bodies Q.NO.12. Give laboratory findings of serum T3, T4 and TSH levels in typical case of Graves disease, Hashimotos thyroiditis; diffuse non-toxic goiter and multinodular goiter. (annual 2007) See Q.NO.4 Q.NO.13. a. name salivary gland tumors. b. describe briefly adenoid cystic carcinoma. (annual 2006) a. Benign Malignant Pleomorphic adenoma Mucoepidermiod carcinoma Warthin tumor Adenocarcinoma Oncocytoma Acinic cell carcinoma Basal cell adenoma Adenoid cystic carcinoma Squamous cell carcinoma Ductal papilloma Malignant mixed tumor c. adenoid cystic adenoma: relatively uncommon 5%. gross appearance: generally small, poorly encapsulated, infiltrative, gray pink lesion histologic: composed of small cell having dark, compact nuclei & scant cytoplasm. The space b/w tumor cells is filled by hyline material. Q.NO.14. Write a note on: renal changes in diabetes mellitus. (Annul 2005) 1. Glomerular lesion: 1. thickening of GBM 2. diffuse glomerulosclerosis 3. nodular glomerularsclerosis 4. exudates lesion (capsuar drops) 2. renal vascular lesion: 1. arteriosclerosis 2. hyline arteriosclerosis of both afferent & efferent arterioles. 3. acute & chroni pyelonephritis 4. diffuse mesangial sclerosis 5. necrotizing papillitis 6. tubular lesion: glycogen infiltrate, glycogen nephrosis or armanni ebstein cells

Edited by Foxit Reader Copyright(C) by Foxit Corporation,2005-2009 For Evaluation Only.

KHIZZER KHAN NMC

Q.NO.15. a. classify the tumors thyroid gland. (supp:2003) b. give the morphological features of medullary carcinoma thyroid. c. cold nodules of thyroid is dangerous. Discuss very briefly. a. tumors of thyroid: benign: 1. follicular thyroid adenoma 2. toxic adenomas carcinomas: 1. papillary carcinoma: 75-85% 2. follicular carcinoma: 5-15% 3. medullary carcinoma: 5% 4. anaplastic carcinomas: <5% b. gross: 1.medullary carcinomas may arise as a solitary nodules or may present as multiple 2. multicentricity is particularly common in familial cases 3. larger lesion often contain areas of necrosis & hemorrhage & may extend to capsule. Light Microscopy: 1. they are composed of polygonal to spindal shaped cells which may form nests, trabeculae, & even follicles. 2. acellular amyloid deposits derived from altered calcitonin molecules are present in adjacent stroma & is distinctive feature of these tumors electron microscopy: 1. reveals variable numbers of intra-cytoplasmic membrane bound electron dense granules 2. one of the peculiar feature is the presence of multicentric C-cell hyperplasia in the surrounding thyroid parenchyma, a feature usually absent in sporadic lesion. d. the cold nodules has high risk of being cancerous or malignant than that of hot nodules so more dangerous c. cold nodules are more likely malignant so dangerous. Q.NO.16. While examining H & E sections from a thyriodectomy specimen, a pathologist notices a lesion having a follicular pattern of growth. What is the differential diagnosis of such lesions? (SZMC. Sendup 2011) b. list the three nuclear features diagnostic for pipallary carcinoma of thyroid.

Edited by Foxit Reader Copyright(C) by Foxit Corporation,2005-2009 KHIZZER KHAN NMC For Evaluation Only.

Q.NO.17. An 18 years old known diabetic girl, suddenly felt ill, developed vomiting and became drowsy. She was taken to the emergency, where on examination she had B.P of 95/60mm of Hg with pulse rate 112/min and cold extremities. She had deep, sighing respiration (kussuauls respiration) and her breath smelt of acetone. (SZMCsendup2011) a. what is the likely diagnosis? b. give any four clinical and four metabolic features of this condition.

KHIZZER KHAN NMC

0300-710-3321

Edited by Foxit Reader Copyright(C) by Foxit Corporation,2005-2009 For Evaluation Only.

Chapter 26

The Muscloskeletal System

KHIZZER KHAN NMC

Q.NO.1.a. Classify the tumors of the bones. (annual 2011) b. give morphology of osteosarcoma. a. Histologic type Benign Malignant Hematopoietic 40% Meyloma Malignant lymphoma Chondrogenic 22% Osteochondroma chondrosarcoma Chondroma Differentiated chondrosarcoma Chondroblastoma Mesenchymal chondrosarcoma Chondromyxoid fibroma Osteogenic 19% Osteod osteoma Osteosarcoma Osteoblastoma Fibrogenic Fibroma Fibrosarcoms Non-ossifying fibroma Fibrous histocytoma Unknown origin 10% Giant cell tumor Unicameral cyst Aneurismal bone cyst Neuroectodermal Ewing sarcoma Notochordal Benign notochordal cell tumor Chordoma b. morphology: gross: gritty, gray-white tumors, often exhibiting hemorrhage and cystic degeneration. frequently destroy the surrounding cortices and produce soft tissue mass. production of mineralized or unmeneralized bone ( osteoid ) by malignant cell. when malignant cartilage is abundant,tumor is called chondroblatic osteosarcoma Microscopy: hyperchromatic nuclei bizarre tumor giant cells more mitosis Q.NO.2.A 45 years old child presents with morning stiffness which started from small joints of hand, and now involves wrist, elbow and knees bilaterally. On examination the joints are found swollen. The x-ray studies show peri-articular osteopenia.(annual2010) a. what is most likely diagnosis? b. give the histological features of this disease. a. osteoarthritis b. earliest structural changes in osteoarthritis include enlargement, proliferation and disorganization of chondrocytes in the superficial part of the articular cartilage. fibrillation and cracking of the matrix: when superficial layer is degraded bone eburnation: friction exposed bone giving appearance of polished ivory joint mice: fracture can dislodge the pieces of cartilage into the joint by forming Loose bodies mushroomed osteophytes : bony outgrowths pannus: this synovial fibrous covers the peripheral portion of the articular surface.

Edited by Foxit Reader Copyright(C) by Foxit Corporation,2005-2009 For Evaluation Only.

Q.NO.3. A 60 years old male presented with pathological fracture of femur. On examination he had multiple punched out lesion in the skull. His serum Ca was 11mg/dl. What further investigations will you do to confirm the diagnosis? Explain your findings in relation to nature of disease. (supp:2010) Multiple myeloma
KHIZZER KHAN NMC

Q.NO.4.A 14 years old boy presents with a one month H/O pain in the knee joint. There is H/O weight loss and pallor but no fever. X-rays of knee joint reveals densely sclerotic lesion in the distal femur extending from the growth plate into the diaphysis. The periosteum is lifted, forming an angle with the cortex. The surrounding soft tissue resembles a sun burst on the radiograph. (annual 2009) a. what is the likely diagnosis? b. enumerate the common sites involved by this tumor. c. give different subtypes of this tumor. a. oteosarcoma b. metaphyseal region of the long bones 60% occurring about the knee 15% around the hip 10% at the shoulder 8% in the jaw c. 1. primary osteosarcoma 2. solitary osteosarcoma 3. intramedulllary osteosarcoma 4. poorly differentiated osteosarcoma on the basis of site of involvement: ( medullary vs cortical) on the basis of degree of differentiation: ( solitary vs multicentric) Q.NO.5.A 30 years old develops excruciating pain in the first metatarsophalangeal joint following a weekend of binge drinking. The pain was associated with localized hyperemia, warmth and tenderness. (Annual 2008). a. what is the likeliest diagnosis and what laboratory tests will you order to confirm the diagnosis? b. describe the morphology of tophus. a. gout lab. Tests: 1. X-ray , serum & urine: uric acid levels, joint aspiration is confirmatory b. Tophi are the pathognomonic hallmarks of the gout formed by large aggregations of urate crystals surrounded by an intense inflammatory reactions of lymphocytes, macrophages and foreign body giant cells, attempting to engulf the mass of crystals. tophi can appear in articular cartilage of joints & peri articular ligaments. superfacial tophi can lead to large ulcerations of the overlying skin Q.NO.6. What is osteoarthritis, and what are the predisposing factors for its development. (Supp:2008)

Edited by Foxit Reader Copyright(C) by Foxit Corporation,2005-2009 For Evaluation Only.

KHIZZER KHAN NMC

The degenerative disease of joint in which degeneration of articular cartilage occur. Primary osteoarthritis: without apparent initiating cause Secondary osteoarthritis: predisposing conditions: previous traumatic injury developmental deformity underlying systemic diseases: a) diabetes b) ochronosis c) hemochromatosis obesity Q.NO.7.a. Describe pathogenesis of gouty arthritis. b. Give salient features of tuberculous osteomyelitis. (annual 2007) a. 1. overproduction of uric acid by de novo synthesis and salvage pathway. 2. reduced excretion of uric acid Secondary causes: 1. increased nucleated cell turnover e.g leukemia 2. decreased renal excretion e.g lead poisoning & alcoholism

Edited by Foxit Reader Copyright(C) by Foxit Corporation,2005-2009 For Evaluation Only.

KHIZZER KHAN NMC

b. bone infection complicates an estimated 1 to 3 % of cases of pulmonary tuberculosis. the organism usually reach the bone through the blood stream & by direct spread with hematogenous spread long bones & vertebrae are favored site lesions are often solitary but can be multicentric in immunocompromised patients. because the tubercle bacillus is microaerophilic, the synovium is common site due to high oxygen pressure. the infection then spread to the adjacent epiphysis, where it cause typical Granulomatous inflammation with caseous necrosis and extensive bone destruction. tuberculosis of vertebral bodies causes vertebral deformity and collapse with Secondary neurologic deficit. Q.NO.8. Name bone forming tumours. Briefly write morphological features of osteogenic sarcoma. (annual 2006) See Q.NO.1 Q.NO.9. Write a note on: Giant cell tumour of bone. (annual 2005). common in 20s-40s of age GCTs are dominated by multinucleated osteoclast type giant cell hence the synonym Osteoclastoma it is benign and locally aggressive current opinion suggests that the giant cell component is likely a reactive macrophage Population & the mononuclear cells neoplastic. Morphology: tumors are large or red brown with frequent cystic degeneration they are composed of uniform oval mononuclear cells with frequent mitosis they are composed of uniform oval mononuclear cells with frequent Mitosis with scattered osteoclast type joint cells containing 100 or more nuclei necrosis, hemorrhage and reactive bone formation are also commonly present. Clinical course: majority of GCTs arise in the epiphysis of the long bones around the knee. frequently cause arthritis like symptoms. occasionally GCTs presents as pathologic fractures radiographically, GCTs are large, purely lytic & eccentric, the overlying cortex is is frequently destroyed, producing a bulging soft tissue mass with a thin shell of of reactive bone. although GCTs are histologically benign, roughly half recur after simple curettage & as many as 4 % metastasize to the lungs. Q.NO.10. a. Classify tumours of bone. (annual 2003) b. define sequestrum and involucrum associated with inflammatory bone disease a. 1. bone forming: benign: Tumors Common locations Age (yr) Morphology Osteoma Facial bone, skull 40-50 Exophytic growths Osteoid osteoma Metaphysis of femur, tibia 10-20 Cortical tumor, interlacing trabecule of bone Osteoblastoma Vertebral column 10-20 Similar to osteoid osteoma Malignant: Primary osteosarcoma Metaphysis of distal femur, tibia 10-20 Growth out ward, lifting periosteum Secondry osteosarcoma Femur, humerus, pelvis >40 Complications of paget disease

Edited by Foxit Reader Copyright(C) by Foxit Corporation,2005-2009 For Evaluation Only.

KHIZZER KHAN NMC 2. cartilaginous: Benign: Osteochondroma Metaphysis of long bones 10-30 Bony excrescences with a cartilaginous cap Chondroma Small bones hand & feet 30-50 Well circumscribed single tumors Malignant: Chondrosarcoma Bones of shoulder, pelvis, femur & ribs 40-60 Arise in medullary cavity

3. Miscellaneous: Giant cell tumor (benign) Epiphysis of long bones 20-40 Lytic lesion that erode cortex Ewing tumor (malignant) Diaphysis & metaphysis 10-20 sheets of round cel contain glycogen b. sequestrum: in osteomyelitis , the lifted periosteum impairs the blood supply to the effected region, and both the suppurative & ischemic injury may cause segmental bone necrosis; the dead piece of bone is called sequestrum. Involucrum: after the first week chronic inflammatory cells become numerous & their release of cytokines stimulate osteoclastic bone resorption, ingrowth of fibrous tissue & deposition of reactive bone in the periphery. When the newly deposited bone forms a sleeve of living tissue around the segment of devitalized infected bone, known as involucrum. Q.NO.11. A 14 years old boy presents with a one month H/O pain in the knee joint. There is H/O weight loss and pallor but no fever. X-rays of knee joint reveals densely sclerotic lesion in the distal femur extending from the growth plate into the diaphysis. The periosteum is lifted, forming an angle with the cortex. The surrounding soft tissue resembles a sun burst on the radiograph. (SZMC. Sendup 2011) a. what is the likely diagnosis? b. enumerate the common sites involved by this tumor. c. give different subtypes of this tumor. Q.NO.12. A 80 years old female develops excruciating pain in the first metatarsophalangeal joint following a weekend of binge drinking. The pain was associated with localized hyperemia, warmth and tenderness. (SZMC sendup 2011) a. what is the likeliest diagnosis? b. what laboratory tests will you order to confirm the diagnosis? c. which type of crystals you will see in this type of arthritis? d. write down another ccrystal deposition disease. a. gout b. uric acid levels joint spiration is confirmatory c. needle shape urate crystals d. 1. urate nephropathy 2. hypertension 3. lead poisoning

Edited by Foxit Reader Copyright(C) by Foxit Corporation,2005-2009 KHIZZER KHAN NMC For Evaluation Only.

Q.NO.13. A woman aged 45 years presented with pain in right proximal femur. Provisional diagnosis made was Pagets disease. (SZMC sendup 2011) a. write down two microscopic findings b. Two lab investigations c. complications a. 1. mosaic pattern 2. annealing of lamellar bone b. 1. x- ray 2. increase serum alkaline phosphatase c. Complications: 1. plethora of skeletal 2. neuromascular abnormalities 3. cardiovascular problems 4. sarcoma

Edited by Foxit Reader Copyright(C) by Foxit Corporation,2005-2009 For Evaluation Only.

KHIZZER KHAN NMC

Chapter 28

The Nervous System

Q.NO.1.A 45 years old lady complains of frequent episodes of seizures, headache and weakness of right arm for which she is very afraid. Her CT scan of the brain is done. The report is not yet out but a provisional diagnosis of space occupying lesion is made. a. what is the most common brain tumor in this age? b. give morphological condition of this kind of tumor. c. what are the major categories of Gliomas? (annual 2011) a. meningiomas b. morphology: c. astrocytoma oligodendroglioma ependymoma Q.NO.2.A 4 years old child presents with a space occupying lesion in the cerebellum. Histological examination shows a poorly differentiated tumor with marked cellularity and sheets of anaplastic cell. (annual 2010) a. what is the most likely diagnosis? b. how does the tumor spread and what is its continuous complication? a. medulloblastoma b. spread through CSF. Often seeds neurisis & invade 4th ventricles. Q.NO.3. A 5 years old boy is evaluated for neuromuscular disorders because of frequent falls and awkward walking. His developmental history is notable for delayed motor skill. Physical examination shows marked muscular weakness of extremities, particularly of proximal muscle group and hypertrophy of calf muscles. (supp:2010) a. what is most likely diagnosis? Give the mode of inheritance/genetic basis of this diseas b. what findings would be likely on muscle biopsy? How is a definitive diagnosis made? Q.NO.4. A 50 years old female has had right sided headache for 5 years, but recently noted mild weakness in her right hand. A CT scan shows a well circumscribed lateral mass compressing the right hemisphere at the frontal-parietal junction.(supp:2010) a. what is the most likely diagnosis? Give the morphology of this neoplasm. b. enumerate the factors that influence the overall prognosis of this tumour.

Edited by Foxit Reader KHIZZER KHAN NMC Copyright(C) by Foxit Corporation,2005-2009 For Evaluation Only.

Q.NO.5.A 6 years old girl develops acute vomiting and neck rigidity. MRI reveals a tumor in the posterior fossa consisting of a large cyst with a nodular mass attach to its wall (cyst with mural nodules). Histologic examination shows elongated astrocytes with long bipolar process and numerous Rosenthal fibers. (annual 2009) a. what is the most likely diagnosis? Enumerate the tumors derived from glial cells. b. name the common poorly differentiated neoplasm of the brain. Give any two general characteristics of this neoplasm. a. pilocystic astrocytoma 1. astrocytoma 2. oligodendroglioma 3. epidymoma b. medulloblastoma 1. common in children 2. tumor occurs exclusively in the cerebellum Q.NO.6. a. List three types of glial tumors. (annual 2008) b. list three of the tumors that are the most commonly metastatic to the brain. a. 1. astrocytoma 2. oligodendroglioma 3. eependyma b. 1. meningioma 2. cancer of peripheral nerves Q.NO.7. Define miningiomas and mention morphological characteristic of it different types. (supp: 2008) Definition: predominantly benign tumors of adults, usually attached to the dura & arising from the meningothelial cells of archnoid & from stromal archnoid cells of the choroid plexus of ventricles. Morphology: Gross: located most commonaly in the front half of the cranial cavity.hemispheric convexity, falx, lesser wing of sphenoid & olfactory groove. appear as irregular bosslated masses that are firmly attached to dura & indent surface of brain. Occasionally growth occurs in plate like fashion producing meningioma en plaque hyperstosis & occasional superficial invasion of overlying bone is also seen. on section, tumor is firm, solid with whorl like pattern. Microscopic: there are three histologic types:

Edited by Foxit Reader Copyright(C) by Foxit Corporation,2005-2009 For Evaluation Only.

KHIZZER KHAN NMC

i. syncytial with prominent cellular whorls & nodules. ii. fibroplastic if composed of spindle shaped bipolar cells iii. transitional with intermediate characteristic & psammoma bodies ( which are spherical., laminated, calcified structures) xanthomatous & myxomatous degeneration & ectopic bone formation can be seen. malignant meningiomas occasionally occur. These resemble fibrosarcomas & may Invade the brain metastasis. Q.NO.8.a. differentiate b/w concussion and contusion injuries to the brain. b. give salient features of cereberovascular disease. (annual 2007) a. concussion: is clinical syndrome of altered consciousness secondary to head injury typically brought about by a change in momentum of head ( when moving head is suddenly arrested by Impact on rigid surface. The characteristic neurological picture includes instantaneous onset of transient neurologic dysfunction , loss of consciousness, temporary respiratory arrest & loss of reflexes. Pathogenesis involves dysregulation of reticular activating system. Contusion: is lesion associated with direct parenchymal injury of the brain either through transmission of kinetic energy to the brain or bruising analogue to what is seen in soft tissue. Contusion is developed due to coup & counter coup injury. b. stroke is clinical term for a disease with acute onset of a neurologic deficit as the results of vascular lesion, either hemorrhage & loss of blood supply. cerebral infarction follows loss of blood supply & can be widespread, focal or affected region with the last robust vascular supply (watershed infarct). focal cerebral infarct are most commonly embolic. primary intraparenchymal hemorrhage are typically due to either hypertension in white & gray matter or cerebral amyloid angiopathy. spontaneous subarachnoid hemorrhages is usually caused by a structural vascular abnormality such as aneurysm or ateriovanous malformation. Q.NO.9.a. Discuss briefly laboratory diagnosis of pyogenic meningitis in childhood. b. give briefly the lab findings of the multiple myeloma. c. write a note on: miningioma. (supp:2003)

Edited by Foxit Reader Copyright(C) by Foxit Corporation,2005-2009 For Evaluation Only.

KHIZZER KHAN NMC

a. 1. blood exams: a. TLC inc. with polymorphonuclear leucocytosis b. ESR increases c. culture positive in 40-60% cases of meningococci, pneumococci. 2. CSF exam 3. other tests: nasal swab or meningococcal culture Q.NO.10. A 70 years old man presented with loss of memory for recent events. He has forgotten his grandchildren names. Also he has lost his way while driving to familiar locations. (SZMC sendup 2011) a. what is most likely diagnosis? b. write down two microscopic features. a. Alzheimer

Please point out mistakes so that mistakes can be omitted before publishing BMMMBBBBBBBBBBBBBBBBBBBBBBBBBBBBBBB Feedback me, E mail: shoaibkanwal@live.com MMMMMMMMMMMMMMMM Contact no: 03434352177 Good luck

KHIZZER KHAN NMC 0300-710-3321

Edited by Foxit Reader Copyright(C) by Foxit Corporation,2005-2009 For Evaluation Only.

KHIZZER KHAN NMC

Q.NO.1. A 56 old male presents with raised voiaceous plaques over his legs & is found to be HIV positive. A biopsy of the lesion revealed a vascular neoplasm. Further investigation revealed lesion with similar histology involving multiple abdominal viscera. a) name the likely tumor? b) list three other form of the same disease? c) which viral DNA do you expect to isolate from this tumor? a. Kaposi sarcoma b. 1. classic or chronic KS (European KS) 2. Lymphadenopathic KS (African KS) 3. Transplant associated KS 4. AIDS associated (epidemic) KS c. human herpes virus Q.NO.2. A 30 years old primaegravida delivers a healthy male baby after a prolonged, obstructed labor. A few hour later she develops respiratory distress with cyanosis, heavy bleeding from vagina and rectum and goes into shock. a. what is your diagnosis? b. what parameter will you monitor in the blood of this patient? c. what are two major triggering mechanism of this condition? Q.NO.3. Give a flow chart outline the pathogenesis of idiopathic pulmonary fibrosis? Diagram: 13.14 medium Q.NO.4. A 65 year old writer with history of 45 packs develops progressive dysphagia to solid & liquids. An endoscopy reveals a 5cm ulcerated mass in the middle esophagus. The esophageal function is normal. a. what type esophageal cancer do your expect to find on histology? b. list the four major categories of risk factors predisposing to this type of cancer with one example of each? a. squamous cell carcinoma b. esophageal disorders: 1. long standing esophagitis 2. achalasia 3. plummer Vinson syndrome life style: 1. alcohol consumption 2. tobacco abuse dietary: 1. deficiency of vitamin A, C, riboflavin 2. deficiency of trace metals ( zinc, molybdenum) 3. fungal contamination of food stuffs

Supplementary 2011

Edited by Foxit Reader Copyright(C) by Foxit Corporation,2005-2009 KHIZZER KHAN NMC For Evaluation Only.

4. high contents of nitrites genetic predisposition : 1. tylosis ( hyperkeratosis of palm & sole) Morphology: polypoid exophytic mass ulceration diffuse infiltrative neoplasms Q.NO.5. Endoscopy in a 55 year old man, known case of GERD, reveals tongues of velvety red mucosa extending into lower esophagus from stomach. a. what is your provisional diagnosis? b. what features would you like for in the biopsy to confirm your diagnosis? c. what is major complication of this lesion & what biopsy feature will help you to predict its risks? a. barrett esophagitis b. 1. esophageal squamous epithelium is replaced by metaplastic columnar epithelium 2. dysplatic changes in the mucosa c. ulcer & stricture, high grade dysplasia predicts the risk of adencarcinoma Q.NO.6. With the help of flow diagram illustrate the possible outcomes in the following case scenarios: a. A carrier of HBV with HDV super infection. b. draw a chart showing serologic markers for hepatitis B virus in acute infection with reolution. Diagram 16.7 & 16.8 Q.NO.7. A 35 year old female who has been treated unsuccessfully for resistant lower urinary tract infection (E.coli) now present with fever, malaise, pain at costovertebral angel & leukocyte castes in the urine. a. what is your diagnosis? b. illustrate with diagram the steps in the pathogenesis of this condition. a. acute pyelonephritis 1. adhesion of bacteria to the mucosal surfaces: a. hematogenous infection: (E.coli, staphylococcus) b. ascending infection: (E.coli, proteus, enterobactor, pseudomonas, klebsiella) 2. instrumentation: catheterization & cystoscopy 3. urinary obstruction: congenital or acquired, (in the stasis of urine bacteria multiply) 4. increased vessicourectal reflex: 5. pregnancy 6. diabetes mellitus

Edited by Foxit Reader Copyright(C) by Foxit Corporation,2005-2009 For Evaluation Only.

KHIZZER KHAN NMC

7. preexisting renal lesion 8. patients sex & age: male & advancing age 9. immunosuppression & immunodifiency Diagram: 14.14

Q.NO.8. In a 70 year old man presenting with testicular mass would you favour: a. a diagnosis of classical or spermatocytic seminoma? b. what morphological features would you confirm your provisional diagnosis? a. spermatocyte seminoma b. gross: soft, pale gray & cut surface reveals mucoid cysts microscopy: contains population of three cells 1. medium sized cells: the most numerous, with round nuclei & eosinophilic cytoplasm 2. smaller cells: with narrow rim of eosinophilc cytoplasm 3. scattered giant cells either uninucleated or multinucleated Q.NO.9. List four major groups of ovarian tumors. Which ovarian tumors are associated with BRCA1 & BRCA2. a. surface epithelial stromal cell tumors: 65-70%, age : 20+ years serous tumor mucinous tumor endometroid tumor clear cell tumor transitional cell carcinoma epithelial stromal (adenosarcoma, malignant mixed mullerian tumor) Germ cell derived tumors: 15-20%, age:0-25+ years teratoma (immature, mature, solid, cystic adenoid, monodermal) dysgerminoma endodermal sinus tumor (yolk sac tumor) choriocarcinoma sex cord stromal tumors: 5-10% : all ages fibroma granulosa theca cell tumor sertoli- leyding cell tumor sex cord tumor with annular tubules gynandroblastoma

Edited by Foxit Reader Copyright(C) by Foxit Corporation,2005-2009 For Evaluation Only.

KHIZZER KHAN NMC steroid ( lipid ) cell tumors metastatic cancer colonic, appendiceal gastric breast Surface epithelial stromal cells are associated with BRCA1 & BRCA2 Q.NO.10. A 55 year old woman presented with an eczema like lesion on her left nipple shows large malignant polygonal cells with pale cytoplasm interspersed with an otherwise normal epidermis of nipple skin. a. what is the diagnosis? b. what type of malignant breast lesion you expect to find underneath the nipple? c. what are various types of underlying breast lesion? a. paget disease b. Ductal carcinoma in situ (DCIS) c. 1. comedocarcinoma 2. solid DCIS 3. cribriform DCIS 4. Papillary DCIS 5. micropapillary DCIS Q.NO.11. A thyroid lesion having follicular architecture is identified in athyroidectomy specimen of a patient with a clinical diagnosis of nodular goiter. a. what is the possible differential diagnosis? b. give your histological features of thyroid adenoma. a. follicular adenoma b. 1. presence of intact, well formed capsule 2. hurthle cell Q.NO.12. A 14 year old boy presents with a mass around the knee joint. X ray shows a large destructive, mixed lytic & blastic mass lifting the periosteum with reactive periosteal bone formation. a. what is most likely diagnosis? b. what is the defining feature of this tumor on histological examination? c. what are other common sites for this tumor? a. osteosarcoma b. bizarre tumor giant cells, formation of the bone by the tumor. c. 15% around the hip 10% at the shoulder 8% in the jaw Q.NO.13. List & give brief account of the three microscopic abnormalities of Alzheimers disease. 1. neuritic (senile) plaques

Edited by Foxit Reader Copyright(C) by Foxit Corporation,2005-2009 For Evaluation Only. KHIZZER KHAN NMC

2. neurofibrillary tangels 3. cerebral amyloid angiopathy 4. granlovacuolar degeneration 5. Hirano bodies Q.NO.14. List two laboratory test each for the evaluation of hepatocyte integrity, biliary excretory function & hepatocyte synthetic function. 1. hepatocyte integrity: cytosolic hepatocellular enzymes serum aspartate aminotransferase (AST) serum alanine aminotransferase (ALT) serum lactate dehydrogenase (LDH) 2. Biliary excretory function: substance normally secreted in bile serum bilirubin: total, direct, delta(covalently linked to albumin urine bilirubin serum bile acids Plasma membrane enzymes serum alkaline phosphatase serum -glutamyl transpeptidase serum 5-nucleotidase 3. hepatocyte function: protein secreted in the blood serum albumin prothrombin time (factor v, vii, x prothrombin, fibrinogen) Hepatocyte metabolism: serum ammonia aminopyrine breath test galactose elimination

Solved by: MUHAMMAD SHOAIB KANWAL


SZMC.RYK

KHIZZER KHAN NMC

You might also like